100
USMLE WORLD STEP 1 PATHOPHYSIOLOGY Question List Pathophysiology Q No: 1 Cardiology Pathophysiology Q No: 42 Endocrinology Pathophysiology Q No: 2 Pulmonology Pathophysiology Q No: 43 Hepatobiliary system Pathophysiology Q No: 3 Pulmonology Pathophysiology Q No: 44 Cardiology Pathophysiology Q No: 4 Cardiology Pathophysiology Q No: 45 Cardiology Pathophysiology Q No: 5 Renal Pathophysiology Q No: 46 Gastrointestinal system Pathophysiology Q No: 6 Hepatobiliary system Pathophysiology Q No: 47 Pulmonology Pathophysiology Q No: 7 Genitourinary Pathophysiology Q No: 48 Hepatobiliary system Pathophysiology Q No: 8 Renal Pathophysiology Q No: 49 Musculoskeletal Pathophysiology Q No: 9 Endocrinology Pathophysiology Q No: 50 Endocrinology Pathophysiology Q No: 10 Endocrinology Pathophysiology Q No: 51 Endocrinology Pathophysiology Q No: 11 Cardiology Pathophysiology Q No: 52 Blood vessels Pathophysiology Q No: 12 Endocrinology Pathophysiology Q No: 53 Cardiology Pathophysiology Q No: 13 Endocrinology Pathophysiology Q No: 54 Endocrinology Pathophysiology Q No: 14 Reproductive system Pathophysiology Q No: 55 Cardiology Pathophysiology Q No: 15 Pulmonology Pathophysiology Q No: 56 Endocrinology Pathophysiology Q No: 16 Pulmonology Pathophysiology Q No: 57 Endocrinology Pathophysiology Q No: 17 Endocrinology Pathophysiology Q No: 58 Cardiology Pathophysiology Q No: 18 Cardiology Pathophysiology Q No: 59 Cardiology Pathophysiology Q No: 19 Blood vessels Pathophysiology Q No: 60 Endocrinology Pathophysiology Q No: 20 Cardiology Pathophysiology Q No: 61 Musculoskeletal Pathophysiology Q No: 21 Endocrinology Pathophysiology Q No: 62 Gastrointestinal system Pathophysiology Q No: 22 Neurology Pathophysiology Q No: 63 Musculoskeletal Pathophysiology Q No: 23 Pulmonology Pathophysiology Q No: 64 Cardiology Pathophysiology Q 24 Pulmonology Pathophysiology Q 65 Cardiology 181

Patho Physio 50q

Embed Size (px)

DESCRIPTION

uworld

Citation preview

Page 1: Patho Physio 50q

USMLE WORLD STEP 1 PATHOPHYSIOLOGY

Question List

Pathophysiology Q No: 1 Cardiology Pathophysiology Q No: 42 Endocrinology

Pathophysiology Q No: 2 Pulmonology Pathophysiology Q No: 43 Hepatobiliary system

Pathophysiology Q No: 3 Pulmonology Pathophysiology Q No: 44 Cardiology

Pathophysiology Q No: 4 Cardiology Pathophysiology Q No: 45 Cardiology

Pathophysiology Q No: 5 Renal Pathophysiology Q No: 46 Gastrointestinal system

Pathophysiology Q No: 6 Hepatobiliary system Pathophysiology Q No: 47 Pulmonology

Pathophysiology Q No: 7 Genitourinary Pathophysiology Q No: 48 Hepatobiliary system

Pathophysiology Q No: 8 Renal Pathophysiology Q No: 49 Musculoskeletal

Pathophysiology Q No: 9 Endocrinology Pathophysiology Q No: 50 Endocrinology

Pathophysiology Q No: 10 Endocrinology Pathophysiology Q No: 51 Endocrinology

Pathophysiology Q No: 11 Cardiology Pathophysiology Q No: 52 Blood vessels

Pathophysiology Q No: 12 Endocrinology Pathophysiology Q No: 53 Cardiology

Pathophysiology Q No: 13 Endocrinology Pathophysiology Q No: 54 Endocrinology

Pathophysiology Q No: 14 Reproductive system Pathophysiology Q No: 55 Cardiology

Pathophysiology Q No: 15 Pulmonology Pathophysiology Q No: 56 Endocrinology

Pathophysiology Q No: 16 Pulmonology Pathophysiology Q No: 57 Endocrinology

Pathophysiology Q No: 17 Endocrinology Pathophysiology Q No: 58 Cardiology

Pathophysiology Q No: 18 Cardiology Pathophysiology Q No: 59 Cardiology

Pathophysiology Q No: 19 Blood vessels Pathophysiology Q No: 60 Endocrinology

Pathophysiology Q No: 20 Cardiology Pathophysiology Q No: 61 Musculoskeletal

Pathophysiology Q No: 21 Endocrinology Pathophysiology Q No: 62 Gastrointestinal system

Pathophysiology Q No: 22 Neurology Pathophysiology Q No: 63 Musculoskeletal

Pathophysiology Q No: 23 Pulmonology Pathophysiology Q No: 64 Cardiology

Pathophysiology Q No: 24 Pulmonology Pathophysiology Q No: 65 Cardiology

Pathophysiology Q No: 25 Endocrinology Pathophysiology Q No: 66 Endocrinology

Pathophysiology Q No: 26 Cardiology Pathophysiology Q No: 67 Cardiology

Pathophysiology Q No: 27 Gastrointestinal system Pathophysiology Q No: 68 Cardiology

Pathophysiology Q No: 28 Gastrointestinal system Pathophysiology Q No: 69 Endocrinology

Pathophysiology Q No: 29 Cardiology Pathophysiology Q No: 70 Endocrinology

Pathophysiology Q No: 30 Cardiology Pathophysiology Q No: 71 Pulmonology

Pathophysiology Q No: 31 Endocrinology Pathophysiology Q No: 72 Pulmonology

Pathophysiology Q No: 32 Blood vessels Pathophysiology Q No: 73 Endocrinology

Pathophysiology Q No: 33 Cardiology Pathophysiology Q No: 74 Genitourinary

Pathophysiology Q No: 34 Cardiology Pathophysiology Q No: 75 Gastrointestinal system

Pathophysiology Q No: 35 Blood vessels Pathophysiology Q No: 76 Endocrinology

Pathophysiology Q No: 36 Cardiology Pathophysiology Q No: 77 Hepatobiliary system

Pathophysiology Q No: 37 Endocrinology Pathophysiology Q No: 78 Endocrinology

Pathophysiology Q No: 38 Endocrinology Pathophysiology Q No: 79 Endocrinology

Pathophysiology Q No: 39 Endocrinology Pathophysiology Q No: 80 Gastrointestinal system

Pathophysiology Q No: 40 Gastrointestinal system Pathophysiology Q No: 81 Gastrointestinal system

Pathophysiology Q No: 41 Endocrinology Pathophysiology Q No: 82 Blood vessels

Pathophysiology Q No: 83 Cardiology Pathophysiology Q No: 101 Blood vessels

Pathophysiology Q No: 84 Cardiology Pathophysiology Q No: 102 Cardiology

181

Page 2: Patho Physio 50q

USMLE WORLD STEP 1 PATHOPHYSIOLOGY

Pathophysiology Q No: 85 Cardiology Pathophysiology Q No: 103 Cardiology

Pathophysiology Q No: 86 Gastrointestinal system Pathophysiology Q No: 104 Cardiology

Pathophysiology Q No: 87 Endocrinology Pathophysiology Q No: 105 Gastrointestinal system

Pathophysiology Q No: 88 Musculoskeletal Pathophysiology Q No: 106 Cardiology

Pathophysiology Q No: 89 Endocrinology Pathophysiology Q No: 107 Cardiology

Pathophysiology Q No: 90 Genitourinary Pathophysiology Q No: 108 Blood vessels

Pathophysiology Q No: 91 Blood vessels Pathophysiology Q No: 109 Cardiology

Pathophysiology Q No: 92 Endocrinology Pathophysiology Q No: 110 Cardiology

Pathophysiology Q No: 93 Endocrinology Pathophysiology Q No: 111 Endocrinology

Pathophysiology Q No: 94 Hepatobiliary system Pathophysiology Q No: 112 Musculoskeletal

Pathophysiology Q No: 95 Pulmonology Pathophysiology Q No: 113 Endocrinology

Pathophysiology Q No: 96 Musculoskeletal Pathophysiology Q No: 114 Hepatobiliary system

Pathophysiology Q No: 97 Gastrointestinal system Pathophysiology Q No: 115 Endocrinology

Pathophysiology Q No: 98 Pulmonology Pathophysiology Q No: 116 Pulmonology

Pathophysiology Q No: 99 Pulmonology Pathophysiology Q No: 117 Endocrinology

Pathophysiology Q No: 100 Cardiology      

182

Page 3: Patho Physio 50q

USMLE WORLD STEP 1 PATHOPHYSIOLOGY

A. Profound hypotension B. Hypertensive emergency C. Left-to-right shunt D. Increased venous return E. Right-to-left shunt

Explanation:The gross autopsy specimen shows a ruptured left ventricular (LV) free wall. This complication of transmural (ST-elevation) myocardial infarction generally occurs 3 to 7 days after the onset of total ischemia, when coagulative necrosis neutrophil infiltration and enzymatic lysis of connective tissue have substantially weakened the infarcted myocardium (mean 4-5 days; range 1-10 days). Free wall rupture causes cardiac tamponade, which greatly limits ventricular filling during diastole. As the pressure increases in the pericardial cavity, venous return to the heart is reduced. This leads to profound systemic hypotension and pulseless electrical activity. Failure to relieve the obstruction will lead to death. Clinically, these patients present with profound hypotension and shortness of breath. On physical examination, the heart sounds are muffled and the jugular venous pressure is elevated. (Choice C) Left-to-right shunting would occur as a result of ventricular septal rupture. (Choice E) Right-to-left shunting is seen in patients with Eisenmenger syndrome, a complication of certain congenital heart diseases. This would be unusual as a complication of Ml.

Educational Objective: The triad of muffled heart sounds elevated jugular venous pressure and profound hypotension indicates pericardial tamponade. Rupture of the ventricular free wall as a consequence of an acute transmural Ml can cause tamponade. Rupture usually occurs 3 to 7 days after the onset of total ischemia, when coagulative necrosis, neutrophil infiltration, and enzymatic lysis of connective tissue have sufficiently weakened the infarcted myocardium.

183

Q NO 1: A 62-year-old Caucasian female hospitalized with acute myocardial infarction dies suddenly on day four of her hospitalization. The autopsy findings are pictured below (RV = right ventricle, LAD = left anterior descending coronary artery). The patient most likely died from which of the following?

Page 4: Patho Physio 50q

USMLE WORLD STEP 1 PATHOPHYSIOLOGY

Explanation:This patient’s clinical picture is consistent with chronic obstructive pulmonary disease (COPD). COPD encompasses chronic bronchitis and emphysema. Heavy smoking is the most common cause. Chronic bronchitis and emphysema have similar effects on FEV1/FVC during pulmonary function testing (PFT). The hallmark of an obstructive PFT profile is decreased FEV1/FVC (FEV1%) due to expiratory airflow obstruction. Emphysema also causes a decrease in EVC and an increase in both TLC and RV due to destruction of interalveolar walls, decrease in lung elastic recoil, and distal airspace enlargement. Choice C is the only option with a decreased (FEV1%) and an increase in both TLC and RV. (Choice E) This PFT profile is characteristic of restrictive lung disease (e.g. pulmonary fibrosis). In restrictive lung disease, lung volumes — particularly TLC and EVC — are decreased due to reduced lung expansion. FEV1/FVC may be increased above the normal value of approximately 80%. This FEV1% increase is the combined result of reduced FVC, decreased lung compliance, and increased elastic recoil.

Educational Objective: Chronic obstructive pulmonary disease (COPD) in a heavy smoker may consist of both emphysema and chronic bronchitis and thus may present with both progressive exertional dyspnea (characteristic of emphysema) and frequent respiratory infections (characteristic of chronic bronchitis). On pulmonary function testing all COPD yields a decreased FEV1/FVC ratio. Emphysema also tends to increase TLC and RV. In contrast, restrictive lung diseases can cause reduced lung volumes and increased FEV1/FVC.

184

Q NO 2: A 65-year-old male presents to your office with exertional dyspnea. He has had four respiratory infections over the course of the past year. For the past 30 years he has smoked 1 ½ packs of cigarettes a day. Physical examination reveals diffusely decreased breath sounds, increased chest anteroposterior diameter, and decreased diaphragmatic excursion. Pulmonary function testing will most likely show which of the following patterns of findings (TLC total lung capacity; FEV 1 forced expiratory volume in 1 second; FVC forced vital capacity; RV, residual volume)?

Page 5: Patho Physio 50q

USMLE WORLD STEP 1 PATHOPHYSIOLOGY

A. Upper airway obstruction B. Poor respiratory drive C. Respiratory muscle fatigue D. Respiratory acidosis E. Alveolar hyperventilation r F. Decreased chest wall compliance

Explanation:This patient has a combination of hypoxemia and hypocapnia. PaCO2 is inversely related to alveolar ventilation, and is considered the main indicator of alveolar ventilation. Assuming a normal rate of metabolic CO2 production, hypocapnia implies alveolar hyperventilation.

PaCO2 = Basal metabolic rate / alveolar ventilationHis hypoxia could be from pulmonary embolism pulmonary edema, pneumonia etc. All these conditions can cause tachypnea resulting in low PaCO2. (Choice A) Significant upper airway obstruction would impair alveolar ventilation and would result in an increase in PaCO2 with a proportionate decrease in Pa02. (Choice B) This patient’s degree of alveolar hyperventilation indicates that his peripheral arterial chemoreceptors sense the hypoxemia and are sending neural impulses to his CNS respiratory centers to increase respiratory drive above normal levels, resulting in hypocapnia. (Choice C) Significant respiratory muscle fatigue would impair alveolar ventilation and would result in an increase in PaCO2. (Choice D) Respiratory acidosis is caused by deficient alveolar ventilation, resulting in an increase in PaCO2 (hypercapnia). (Choice F) A decrease in chest wall compliance could increase the work of breathing and thereby result in respiratory muscle fatigue. Alveolar hypoventilation and increased PaCO2 with a proportionate decrease in PaO2 could result.

Educational Objective: Arterial PaCO2 is a direct indicator of the status of alveolar ventilation. Hypocapnia implies ongoing alveolar hyperventilation. Upper airway obstruction, reduced ventilatory drive, respiratory muscle fatigue, and decreased chest wall compliance are possible cause alveolar hypoventilation, which would cause hypercapnia.

185

Q NO 3: A 45-year-old male presents to the ER with severe dyspnea of recent onset. He says he has never experienced symptoms like this before. Arterial blood gases show a Pa02 of 54 mmHg and a PaCO2 of 26 mmHg. The process most likely responsible for this patient’s condition is:

Page 6: Patho Physio 50q

USMLE WORLD STEP 1 PATHOPHYSIOLOGY

A. AB. BC. CD. DE. E

Explanation:Cardiac catheterization shows a hemodynamic profile consistent with aortic regurgitation (AR). Note the high peaking left ventricular and aortic pressures during systole and the steep diastolic decline in aortic pressure. A normal catheterization report is shown below for purposes of comparison:

The peak intensity of an AR murmur occurs after closure of the incompetent aortic valve, at the point when the pressure gradient between the aorta and the left ventricle is at its maximum i.e. time C. (Choice A) This time point corresponds to the opening of the aortic valve during systole. The murmur of aortic stenosis would be heard best here. (Choice B) This point corresponds to the closure of the aortic valve. The A2 heart sound is heard here. At this instant there is not yet regurgitant flow from the aorta to the left ventricle, so no murmurs are audible.

186

Q NO 4: A 52-year-old Caucasian male presents to your office with two week history of progressive fatigue and exertional dyspnea. He brings with him the report from a recent cardiac catheterization (shown below). Cardiac auscultation reveals a murmur that is best heard when the patient sits up and leans forward. Which of the time points pictured below corresponds to the peak murmur intensity?

Page 7: Patho Physio 50q

USMLE WORLD STEP 1 PATHOPHYSIOLOGY

(Choice D) Time point D occurs in mid-diastole. The murmur of AR might be heard here, as there is a pressure gradient between the aorta and left ventricle (LV). However the intensity of the murmur would be less than at time C because the magnitude of the gradient is less. Because the AR murmur decreases in intensity with the falling aortic pressure, it is a “decrescendo” diastolic murmur. (Choice E)Time E marks the onset of left atrial contraction at the end of ventricular diastole. If the murmur of AR were still audible at this time, its intensity would be further reduced by the increase in left ventricular end diastolic pressure.

Educational Objective: The murmur of AR is a diastolic decrescendo murmur heard loudest in early diastole when the pressure gradient between the aorta and the left ventricle is maximal.

187

Page 8: Patho Physio 50q

USMLE WORLD STEP 1 PATHOPHYSIOLOGY

A. Proximal tubules due to high solute concentration B. Proximal tubules due to impaired uric acid transport C. Loop of Henle due to urine hyposmolarity D. Distal tubules due to high urine flow rate E. Collecting ducts due to low urine pH

Explanation:Tumor lysis syndrome is an oncologic emergency. It often develops during chemotherapy for high-grade lymphomas, leukemias, and other tumors that have rapid cell turnover and high sensitivity to chemotherapy. When a large number of tumor cells are destroyed during chemotherapy, intracellular ions, such as potassium, phosphorous, and uric acid (a metabolite of tumor nucleic acid), are released into the serum and are then filtered by the kidneys. Uric acid (pKa = 5.4) is soluble at physiologic pH, but precipitates in an acidic environment. The lowest pH along the nephron is found in the distal tubules and collecting ducts; so these are the segments of the nephron that become obstructed by uric acid crystals. Obstructive uropathy and acute renal failure follow. The risk of tumor lysis syndrome can be reduced by urine alkalinization and hydration. Additionally allopurinol (a xanthine oxidase inhibitor) is used to reduce uric acid production during the breakdown of tumor cells. (Choice D) Ignore the anatomy portion of choice D and evaluate the latter portion. A “high urine flow rate” would universally decrease uric acid crystallization and precipitation. Therefore, this cannot possibly be the correct answer. (Choices A, B and C) Uric acid does not precipitate in proximal tubules or in Henle’s loop.

Educational Objective: Tumor cell syndrome occurs when tumors with a high cell turnover are treated with chemotherapy. The lysis of tumor cells causes intracellular ions such as potassium and phosphorous, and uric acid (metabolite of tumor nucleic acid) to be released into serum. Uric acid is soluble at physiologic pH, but it can precipitate in the normally acidic environment of distal tubules and collecting ducts. The prevention of tumor lysis syndrome includes urine alkalinization and hydration, as high urine flow and high pH along the nephron prevents crystallization and precipitation of uric acid.

188

Q NO 5: A 34-year-old male who is being treated for acute leukemia develops oliguria. His serum creatinine level is 2.7 mg/dL. Renal biopsy reveals multiple uric acid crystals obstructing renal tubular lumen. The principal site of uric acid precipitation would be which of the following?

Page 9: Patho Physio 50q

USMLE WORLD STEP 1 PATHOPHYSIOLOGY

A. Occlusion of the middle cerebral artery B. Accumulation of blood urea nitrogen C. Increased absorption of nitrogenous substances from gut D. Decreased concentrations of y-aminobutyric acid (GABA) E. Bacterial infection of the meninges

Explanation:Hepatic encephalopathy is a reversible decline in neurologic function precipitated by hepatic damage. The pathogenesis of this condition is likely related to increased levels of ammonia in circulation which cause inhibitory neurotransmission via the GABA receptors in the central nervous system. Ammonia initially enters circulation through the gastrointestinal tract, after having been created during the enterocytic catabolism of glutamine and the bacterial catabolism of dietary protein in the colon. The ammonia then enters the liver through the portal vein for detoxification to urea. Because the damaged liver has impaired detoxification ability however, ammonia accumulates in the blood instead. Frequently, hepatic encephalopathy is precipitated by a stressor that alters the ammonia balance (eg, hypovolemia, gastrointestinal bleeding hypokalemia, metabolic alkalosis, hypoxia, sedative usage, hypoglycemia, or infection). Lowering of the blood ammonia level is typically accomplished with continuous administration of a disaccharidase such as lactulose. Bacterial action on the lactulose results in acidification of colonic contents, which converts the absorbable ammonia into nonabsorbable ammonium ion (an ammonia trap). (Choice A) The middle cerebral artery is the largest cerebral artery and is most commonly involved in cerebrovascular accidents (CVAs). This patient’s presentation is suggestive of hepatic encephalopathy, however, and not stroke. (Choice B) Accumulation of blood urea nitrogen is suggestive of renal failure, heart failure, or dehydration. Liver disease is associated with decreased blood urea nitrogen because less ammonia is converted to urea. (Choice D) Increased (not decreased) activity of the GABA neurotransmitter system is thought to be directly responsible for the altered mental status seen in hepatic encephalopathy. (Choice E) Bacterial infection of the meninges is characteristic of meningitis which does not fit this patient’s presentation as well as hepatic encephalopathy does.

Educational Objective: Hepatic encephalopathy appears to be secondary to increased levels of ammonia in circulation which cause inhibitory neurotransmission via the GABA receptors in the central nervous system. Frequently hepatic encephalopathy is precipitated by a stressor that alters the ammonia balance (eg, gastrointestinal bleeding).

189

Q NO 6: A 54-year-old known alcoholic is brought to the emergency room because of hematemesis. By the following morning he has developed altered mental status. Physical examination shows abdominal distention, flapping tremor, and gynecomastia. Liver span is decreased. Which of the following is the most likely cause of his altered mental status?

Page 10: Patho Physio 50q

USMLE WORLD STEP 1 PATHOPHYSIOLOGY

A. Increased ionized calcium concentration B. Increased ionized phosphate concentration C. Decreased free water D. Acidification of the urine E. Saturation with uric acid F. Saturation with citrate

Explanation:Urine is a complex solution with a large number of compounds in a dynamic balance. Changes to concentrations and ratios of components such as calcium, phosphate oxalate, uric acid, and citrate can shift this balance to cause salt precipitation and stone formation. Increased excretion of stone-forming compounds can cause urine supersaturation. When fluid intake is low, the concentrations of these ions in urine are increased (without affecting absolute amounts). When supersaturation occurs precipitation and aggregation of crystals follows. The crystal mass may attach to the surface of the renal papillae, facilitating the continued aggregation of salts around this nidus. High fluid intake decreases the concentrations of stone-forming ingredients, thus preventing stone formation. A high urine citrate concentration has a stone-preventing effect as well. Citrate binds to free (ionized) calcium, preventing its precipitation and facilitating its excretion. (Choices A, B and C) Increasing ionized calcium and ionized phosphate concentrations, and decreasing water intake will all promote urine supersaturation. (Choice D) Low urine pH is required for the formation of uric acid and cystine stones. Calcium salts can precipitate at acidic as well as neutral pHs. Thus, urine acidification would encourage crystals to precipitate in this solution. (Struvite stones are unique in that they occur at an alkaline pH.) (Choice E) Hyperuricosuria is associated with a number of conditions, including gout myeloproliferative disorders, and situations of high purine and alcohol intake. It can also be idiopathic. Increased urine uric acid concentration promotes the formation of calcium and uric acid stones.

Educational Objective: Renal calculi occur when there is an imbalance of the factors that facilitate and prevent stone formation. Increased concentrations of calcium, phosphate, oxalate, and uric acid promote salt crystallization, whereas increased citrate and high fluid intake help prevent calculi formation.

190

Q NO 7: Which of the following interventions would be most likely to increase the threshold for crystal precipitation in a solution of calcium, phosphate and oxalate salts?

Page 11: Patho Physio 50q

USMLE WORLD STEP 1 PATHOPHYSIOLOGY

A. Afferent arterioles B. Afferent arterioles C. Vasa recta D. Distal tubules E. Proximal tubules

Explanation:Enalapril is an ACE-inhibitor. All ACE-inhibitors decrease the amount of circulating angiotensin II, a substance that causes: a) systemic vasoconstriction, b) preferential constriction of the glomerular efferent arteriole, and c) enhancement of adrenal cortical aldosterone secretion. Thus, reduction of available angiotensin II by an ACE-inhibitor would be expected to acutely decrease systemic vascular and efferent arteriolar resistance. Selective efferent arteriolar dilation and decreased systemic vascular resistance both have the effect of reducing the GFN.

Educational Objective: In the kidney, angiotensin II preferentially constricts the efferent arteriole, thereby maintaining the GER. ACE-inhibitors promote efferent arteriolar dilation, causing GFN reductions.

191

Q NO 8: A 44-year-old male presents to your office for a routine check-up. His past medical history is significant for mild hypercholesterolemia that he has been able to control through diet. His father died of a myocardial infarction at the age of 56, and his mother, who is still living, has a history of stroke. On physical examination, the patient has a blood pressure of 160/100 mmHg and a heart rate of 70 beats per minute. You start him on enalapril. Over the first several days of therapy, the patient’s glomerular filtration rate (GER) adjusts in the following fashion: Enalapril’s effects on which of the following structures is most likely responsible for this renal response

Page 12: Patho Physio 50q

USMLE WORLD STEP 1 PATHOPHYSIOLOGY

A. Osteoporosis B. Gastric ulcers C. Myocardial infarction D. Stroke E. Rapid enlargement of adenoma

Explanation:High levels of prolactin suppress gonadotropin-releasing hormone (GnRH) secretion from the hypothalamus, leading to hypogonadism. (High levels of prolactin in females can also cause milk discharge from the breasts, known as galactorrhea.) As hyperprolactinemia causes hypogonadism, ie low estrogen in females, affected patients are at risk for accelerated bone loss. Estrogens maintain bone mass in females so any loss of estrogen—whether from menopause, hormone imbalances, or surgical removal of the ovaries—leads to loss of bone density. Severe loss of bone density is described by the word “osteoporosis.” (Choice B) Multiple endocrine neoplasia (MEN) type I consists of the triad of hyperparathyroidism, hypergastrinemia, and pituitary adenoma. A good mnemonic is to remember 3Ps: para thyroidism (hyper), peptic ulcer due to a gastrin secreting tumor (usually in the pancreas), and pituitary adenoma. If this patient did have the MEN syndrome, her family history would most likely have some “red flags.” The chances of this patient developing a gastric ulcer are no higher than the general population’s. (Choices C and D) Patients with hyperprolactinemia are not at increased risk for myocardial infarction or stroke. Estrogen was once thought to be cardioprotective, but that is now a very controversial belief. (Choice E) Most prolactinomas in females are smaller than 10mm microprolactinoma. Most males present with macroprolactinoma (tumor larger than 10mm in size) because men typically do not have symptoms until a pituitary tumor is very large. Even without treatment, the risk of rapid enlargement of a microadenoma is low.

Educational Objective: Hyperprolactinemia causes hypogonadism, which leads to reduced estrogen in women. Low estrogen due to any cause is risk factor for accelerated bone loss.

192

Q NO 9: A 40-year-old female presents with amenorrhea. Her family history is unremarkable. Labs reveal an increased prolactin level. MRI shows a 6-mm pituitary adenoma. The patient refuses medical and surgical therapy, as she is happy about not having menstrual period. An untreated prolactin secreting pituitary adenoma puts this patient at greatest risk of developing which of the following?

Page 13: Patho Physio 50q

USMLE WORLD STEP 1 PATHOPHYSIOLOGY

Serum sodium 120 mEq/L Serum potassium 5.6 mEq/L Chloride 90 mEq/L Bicarbonate 6 mEq/L Blood glucose 60 mg/dL

Abdominal imaging shows bilateral adrenal hyperplasia. Further evaluation will most likely show?

Explanation:

193

Q NO 10: Soon after birth, a neonate develops vomiting and hypotension. Physical examination shows clitoromegaly. Laboratory studies show: Chemistry panel

Page 14: Patho Physio 50q

USMLE WORLD STEP 1 PATHOPHYSIOLOGY

The patient described in the vignette has clinical features of cortisol deficiency (hyponatremia, hyperkalemia, acidosis and hypoglycemia) as well as androgen excess (clitoromegaly). Her pattern of laboratory abnormalities in combination with the results of her abdominal imaging point to a diagnosis of congenital adrenal hyperplasia (CAH). CAH encompasses a group of disorders that stem from various defects in the enzymes involved in cortisol biosynthesis by the adrenal gland. The result is an increase in cortisol precursors proximal to the enzyme deficiency. The specific pattern of precursor excess can be used to make the biochemical diagnosis of these disorders. Deficiency of 21-hydroxylase is the most common cause of CAH, accounting for 90% of patients. This enzyme is responsible for the conversion of 17 hydroxyprogesterone to 11-deoxycortisol in the zona fasciculata, and for the conversion of progesterone to deoxycorticosterone in the zona glomerulosa. Thus, serum 1 7-hydroxyprogesterone levels are elevated in this condition because the enzymatic blockade prevents its conversion to 11-deoxycorlisol. As a result of this enzyme deficiency, the adrenal gland cannot synthesize cortisol efficiently. This causes an increased production of adrenal androgens, because the accumulating cortisol precursors are diverted towards the adrenal androgen biosynthetic pathway. The resultant low cortisol levels stimulate pituitary production of ACTH, which increases the production of adrenal androgens even further.

Educational Objective: Deficiency of 21-hydroxylase is the most common type of congenital adrenal hyperplasia. Patients with classic 21- hydroxylase deficiency present with clinical manifestations of cortisol and aldosterone deficiency combined with androgen excess. (The genitalia of female infants maybe masculinized to some degree; male infants, however, are normal in appearance.)

194

Page 15: Patho Physio 50q

USMLE WORLD STEP 1 PATHOPHYSIOLOGY

Left Ventricular Left Ventricular Left Ventricular Cavity Ejection Fraction End-Diastolic

Pressure A. Dilated Decreased Increased B. Dilated Decreased Normal C. Normal Decreased Decreased D. Normal Decreased Increased E. Normal Normal Increased F. Dilated Normal Normal

Explanation:This patient has symptoms of heart failure, defined broadly as a pathophysiological state wherein the heart either cannot pump enough blood to meet tissue metabolic requirements, or can do so only from an elevated ventricular filling pressure. Heart failure may be systolic and/or diastolic. Diastolic heart failure designates a pathologic reduction in diastolic ventricular compliance. Left ventricular end-diastolic volume (LVEDV) and therefore stroke volume and cardiac output are reduced at normal filling pressures (LVEDP). The Frank-Starling curve relating stroke volume to LVEDV, and therefore ejection fraction (EF)I is normal. LVEDP must be increased to abnormally high values to achieve a normal LVEDV and thereby restore cardiac output to near normal. Left ventricular systolic failure implies that stroke volume and cardiac output are reduced at a normal LVEDV. The Frank—Starling curve relating stroke volume to LVEDVI and therefore EF, is depressed. To maintain a near normal cardiac output, both LVEDV and thus LVEDP must be abnormally increased. In summary, LVEDP must be abnormally increased to restore cardiac output in both systolic and diastolic heart failure. However, LVEDV remains normal in diastolic failure but is increased in systolic failure. Thus we can eliminate Choices A, B, and F. Choice E is correct because it indicates a normal LVEDV and a normal ventricular performance (EF) with an elevated LVEDP. Isolated diastolic failure may result from hypertrophic or restrictive cardiomyopathy. (Choice A) This choice characterizes isolated systolic left ventricular failure which could result from an acute massive myocardial infarction, for example. (Choices B and C) These choices describe states with systolic left ventricular failure (reduced EF) and increased diastolic left ventricular compliance. (Choice D) This option represents a state with both decreased left ventricular contractile performance (EF) and decreased diastolic left ventricular compliance indicating a combination of both systolic and diastolic left ventricular failure, which may be seen in chronic ischemic heart disease. (Choice F) This choice describes a state with normal left ventricular contractile function (EF), but increased diastolic left ventricular compliance.

195

Q NO 11: A 45-year-old Caucasian male presents to your office with exertional dyspnea and easy fatigability. He has not seen a physician for 10 years and has no knowledge of any medical problems. He does not take any medications. and admits to cigarette smoking and alcohol use. His BP is 170/90 mmHg and his heart rate is 80 beats per minute. There are bilateral lung crackles on physical exam. Which of the following set of laboratory findings would be most consistent with diastolic heart failure in this patient?

Page 16: Patho Physio 50q

USMLE WORLD STEP 1 PATHOPHYSIOLOGY

Educational Objective: Diastolic heart failure is characterized by normal ventricular contractile performance (EF) but a decrease in ventricular diastolic compliance. As a result, ventricular end-diastolic pressure (EDP) must be increased to achieve a normal ventricular end-diastolic volume (EDV) and stroke volume. Systolic heart failure is a decrease in ventricular contractile performance which requires increases of ventricular EDV and therefore also EDP to achieve a normal stroke volume. In summary, diastolic failure increases only FDP, whereas systolic failure increases both EDP and FDV.

196

Page 17: Patho Physio 50q

USMLE WORLD STEP 1 PATHOPHYSIOLOGY

A. Hypertrophy of the glomerular layer of the cortex B. Hypertrophy of the fasciculate layer of the cortex C. Hyperplasia of the adrenal medulla D. Hyperplasia of the fasciculate layer of the cortex E. Diffuse atrophy of the cortex

Explanation:The adrenal gland is separated into an outer cortex and an inner medulla. The outer cortex is further divided into three zones: the zona glomerulosa, the zona fasciculata, and the zona reticularis. The zona glomerulosa contains cells that secrete mineralocorticoid hormones (primarily aldosterone). The zona fasciculata is the broadest of the three zones and contains cells that secrete glucocorticoid hormones (primarily cortisol). Lastly, the zona reticularis contains cells that secrete small amounts of androgens. Individuals who have ACTH-secreting pituitary adenomas gradually develop diffuse hyperplasia of the adrenocortical zona fasciculata a condition termed Cushing’s syndrome. Early in the course of the disease the ACTH causes increased adrenal blood flow. The conversion of cholesterol to delta-5-pregnenolone (the initial, rate-limiting step in cortisol synthesis) is increased. As time passes, the high levels of ACTH increase the total RNA and protein synthesis, as well as the amount of DNA present and the adrenal weight. The enzymes in the steroidogenic pathway are produced in larger amounts as well. (Choice A) Hypertrophy of the glomerular layer of the cortex is associated with Conn’s syndrome, a condition characterized by increased aldosterone secretion. (Choice B) While hypertrophy of the fasciculate layer of the cortex may occur to a limited extent in patients with Cushing’s syndrome, the primary histologic feature of this condition is hyperplasia of the zona fasciculata. (Choice C) Hyperplasia of the adrenal medulla is nota common concern. The pathology more often observed in the adrenal medulla includes pheochromocytoma (associated with excessive adrenaline and noradrenaline production) or neuroblastomas (highly malignant embryonal tumors that present in childhood). (Choice E) Diffuse atrophy of the cortex is associated with Addison’s disease, an autoimmune condition in which the adrenal glands become markedly atrophic. The adrenal medulla is spared.

Educational Objective: Prolonged ACTH stimulation causes hyperplasia of the adrenocortical zona fasciculata, resulting in excessive cortisol production (Cushing’s syndrome).

197

Q NO 12: A 34-year-old female with a recently diagnosed intrasellar microadenoma presents with recent weight gain, fatigue and hypertension. Plasma ACTH levels are persistently high on repeated measurements. Which of the following pathologic changes in the adrenal glands are most likely responsible for this patient’s symptoms?

Page 18: Patho Physio 50q

USMLE WORLD STEP 1 PATHOPHYSIOLOGY

A. Small intestine B. Liver C. Pancreas D. Skeletal muscles E. Adrenals F. Adipose tissue G. Kidney

Explanation:Glucagon increases serum glucose by increased production of glucose from the liver. This is achieved by increasing glycogenolysis (breakdown of glycogen) and increase in gluconeogenesis (production of glucose from non- carbohydrate sources). (Choice C) Glucagon stimulates insulin secretion from the pancreas. However, patients with type 1 diabetes typically do not have residual beta cells. Therefore, glucagon will not have a significant effect on the pancreas of type 1 diabetics. (Choices D, E and F) Epinephrine increases glucose by multiple mechanisms, including increased glycogenolysis and gluconeogenesis in the liver. In skeletal muscle, epinephrine decreases glucose uptake. Epinephrine also causes increased alanine release from skeletal muscle, which serves as a source of gluconeogenesis in the liver. In adipose tissue, epinephrine increases the breakdown of triglycerides thereby increasing free fatty acids and glycerol in the circulation: these can be utilized as gluconeogenetic substrates as well. Glucagon has insignificant effect on skeletal muscle cells and adipocytes. (Choice G) During first 24-hours of fasting the liver is the main organ responsible for providing glucose. When hypoglycemia is sustained gluconeogenesis in the kidneys becomes an important source. Glucagon does not have any substantial effect on gluconeogenesis in the kidneys.

Educational Objective: Glucagon increases serum glucose by increased production of glucose from the liver. Glucagon stimulates insulin secretion from the pancreas. However, patients with type 1 diabetes rarely have significant residual beta cells. Unlike epinephrine, glucagon has an insignificant effect on skeletal muscle cells and adipocytes.

198

Q NO 13: A 24-year-old male who was diagnosed with diabetes two years ago temporarily loses consciousness after he skipped a meal that was to follow his insulin injection. His girlfriend administered glucagon immediately, as instructed by the physician and the patient recovered consciousness in ten minutes. Metabolic changes in which of the following organs are mostly responsible for this patient’s recovery?

Page 19: Patho Physio 50q

USMLE WORLD STEP 1 PATHOPHYSIOLOGY

A. Low plasma TSH B. Low

plasma androstenedione C. Low plasma estradiol D. Increased plasma ESH E. Increased plasma prolactin

Explanation:Klinefelter syndrome is characterized by a karyotype with two or more X chromosomes (47XXY is present in 82% of all cases). It is one of the most common causes of male hypogonadism, reduced spermatogenesis, and male infertility. Histologic examination of the test is in these patients reveals some or all of the testicular tubules to be completely atrophied and replaced by pink hyalinized tissue. Afflicted individuals demonstrate a distinctive body habitus of an elongated body with abnormally long legs, small atrophic testes and small penis, and absent secondary male characteristics (including deep voice, beard, and male pattern pubic hair). Gynecomastia and a mildly decreased 10 are common. Laboratory findings include consistent elevation of plasma gonadotropins (primarily follicle-stimulating hormone) and estradiol, with a reduction in testosterone. The estrogen: testosterone ratio determines the extent of feminization. (Choice A) Significant variation in thyroid stimulating hormone levels is nota classic finding in Klinefelter syndrome. (Choice B) Significant variation in plasma androstenedione (an intermediate step in the biochemical synthesis of testosterone or the estrogens estrone and estradiol) is not a classic finding in Klinefelter syndrome. (Choice C) Estradiol is elevated, not decreased, in patients with Klinefelter syndrome. (Choice E) While elevated prolactin levels can cause gynecomastia, they are not classically associated with Klinefelter syndrome.

Educational Objective: Increased plasma follicle-stimulating hormone (ESH) reflects gonadal failure in patients with Klinefelter syndrome. The estrogen: testosterone ratio determines the extent of feminization.

199

Q NO 14: A 23-year-old Caucasian male is evaluated for bilateral breast enlargement. He has a tall stature and little body hair. His testicles are small and Hair. Which of the following findings would you expect most on laboratory evaluation?

Page 20: Patho Physio 50q

USMLE WORLD STEP 1 PATHOPHYSIOLOGY

A. Serum osmolarity B. Serum sodium C. Serum ketones D. Urine glucose E. Urine chloride

Explanation:Serum pH greater than 7.40 indicates alkalosis. This acid-base disturbance may occur due to decreased pCO2 (respiratory alkalosis) or due to a relative increase in the concentration of HCO3 (metabolic alkalosis). Respirator alkalosis is characterized by a high pH, a low pCO2 and a compensatory decrease in HCO3. Metabolic alkalosis is associated with a high pH, a high HCQ3 and a compensatory increase in pCO2. High pCO2 in association with an increased pH in the patient described in the question stem is suggestive of metabolic alkalosis. The most common causes of metabolic alkalosis are: 1. Loss of hydrogen ions from the body: Vomiting and nasogastric suction cause loss of hydrochloric acid present in gastric secretions. This causes the serum chloride to decrease leading to a decrease in urinary chloride to less than 10 mEq/L. Such metabolic alkalosis is called saline-responsive. It is associated with volume loss and can be corrected by volume repletion with isotonic saline. 2. Thiazide and loop diuretics increase renal losses of Na, which is followed by excretion of Cl. Reabsorption of HCO3 increases to maintain electric neutrality in the cells. The volume contraction caused by diuretics stimulates increased aldosterone secretion, and aldosterone acts to resorb sodium and water from the distal tubule while wasting potassium and hydrogen in urine. Urinary Cl concentration during diuretic therapy is increased; however, metabolic alkalosis associated with diuretic use is chloride-responsive. The overall chloride concentration in the body is low due to increased renal losses, and administration of saline improves acid-base status. This is known as contraction alkalosis. 3. The increased aldosterone secretion seen in primary hyperaldosteronism (Conn syndrome) is also associated with metabolic alkalosis. Aldosterone increases renal Na reabsorption and urinary losses of K, Cl and H with a relative increase in HCO3 resulting from H losses. The urinary Cl concentration is increased (20 mEq/L) in these cases, but administration of chloride does not correct the alkalosis (saline-resistant metabolic alkalosis). Checking the urine chloride (choice C) and ascertaining the patient’s volume status is an important step in the workup of metabolic alkalosis. (Choices A and B) Serum sodium and osmolarity are useful for evaluation of hyponatremia. (Choices C and D) Measurement of urinary and serum glucose and ketones is helpful for the evaluation of metabolic acidosis. Educational Objective: Metabolic alkalosis is characterized by a high arterial blood pH, HCO3 and pCO2. It is most commonly caused by vomiting, NO suction, diuretic use or hyperaldosteronism. Measuring the urinary chloride concentration and determining the patients volume status helps to identify the cause of metabolic alkalosis.

200

Q NO 15: A 35-year-old male who works as a nurse in local hospital is brought to the emergency room with confusion and lethargy. His temperature is 36.7C (98E), blood pressure is 86/48 mm Hg, pulse is 120/mm, and respirations are 12/mm. Arterial blood gas reveals pH 7.59, pC0249 mmHg and p0285 mmHg. Which of the following is most useful in diagnosing the cause of this patient’s condition?

Page 21: Patho Physio 50q

USMLE WORLD STEP 1 PATHOPHYSIOLOGY

A. Increased chloride secretion B. Increased sodium absorption C. Intracellular potassium depletion D. High bicarbonate transport rate E. High mucus water content

Explanation:Recurrent otitis media and sinusitis in a young Caucasian should raise suspicion for cystic fibrosis, as these infections can be caused be the secretion of abnormally thick mucus by the paranasal sinuses and middle ear epithelium. The diagnosis of cystic fibrosis (CF) is usually based on high sweat chloride concentrations, characteristic clinical findings (including sinopulmonary infections) and/or family history. However, a small portion of patients with CF, especially those with a “mild” mutations of the CF transmembrane regulator ion channel (CFTR), have near-normal sweat tests (sweat chloride <60 Mm/L). In these cases, measuring the nasal transepithelial potential difference in vivo can be a diagnostic adjunct. Individuals with CF have a significantly more negative baseline nasal potential difference than normal, due to abnormalities in ion and water transport in the apical luminal membrane of exocrine and mucous gland ductal epithelia. The figure below illustrates these abnormalities, applicable to most exocrine glands, but not sweat glands. (In sweat glands the tissue-specific effect of the CFTR mutation on electrolyte transport is different.)

Here we see that the abnormal CFTR reduces ductal epithelial chloride secretion and increases sodium and water resorption. The result is dehydrated mucus and a widened transepithelial potential difference.

201

Q NO 16: A 6-year-old Caucasian male with recurrent otitis media and sinusitis is found to have a higher than normal nasal transepithelial potential difference. Which of the following processes most likely underlies this finding?

Page 22: Patho Physio 50q

USMLE WORLD STEP 1 PATHOPHYSIOLOGY

(Choice C) CFTR mutations do not dramatically alter transmembrane potassium transport or homeostasis. (Choice D) CFTR mutations can impair transmembrane bicarbonate conductance and lower the rate of exocrine duct bicarbonate secretion, promoting mucin precipitation particularly in pancreatic ducts. A widened transepithelial voltage gradient across the nasal mucosal epithelium would be more likely to involve increased sodium absorption than a high rate of bicarbonate transport.

Educational Objective: In cystic fibrosis abnormalities of the CFTR transmembrane protein (in exocrine glands other than sweat glands) reduce ductal epithelial chloride secretion and increase sodium and net water reabsorption, resulting in dehydrated mucus and a widened transepithelial potential difference.

202

Page 23: Patho Physio 50q

USMLE WORLD STEP 1 PATHOPHYSIOLOGY

A. Lung cancer B. Adrenal adenoma C. Adrenal malignancy D. Pituitary adenoma E. Exogenous glucocorticoid intake

Explanation:This patient’s clinical presentation is consistent with Cushing syndrome, the syndrome of glucocorticoid excess. Causes of Cushing syndrome include: pharmacological doses of exogenous glucocorticoids (commonest cause) ACTH-secreting pituitary adenoma, ectopic production of ACTH or CRH, primary adrenocortical hyperplasia or adrenocortical adenoma. Of these only pituitary adenoma and ectopic ACTH syndrome will have elevated ACTH. The other causes will have suppressed serum ACTH levels. Cushing syndrome that results from an ACTH secreting pituitary microadenoma is termed Cushing’s disease. The screening tests for endogenous Gushing syndrome include overnight low-dose dexamethasone suppression test and 24-hour urine free cortisol. Administration of dexamethasone, a potent glucocorticoid, should suppress AGTH and cortisol levels in normal individuals. However, patients with endogenous Cushing syndrome do not suppress serum cortisol levels following administration of low-dose dexamethasone. Typically, in patients with Cushing’s disease cortisol levels do not suppress with low-dose dexamethasone but do suppress with high-dose dexamethasone. This test is useful in differentiating Cushing’s disease from Cushing syndrome caused by ectopic ACTH production. When there is an ectopic source of ACTH, ACTH levels are typically markedly elevated, and there is no suppression of ACTH or cortisol with even high-dose dexamethasone. (Choice A) Ectopic ACTH production may be seen with small cell lung cancer. Serum ACTH levels are generally markedly elevated in ectopic ACTH secretion by malignant tumors. High-dose dexamethasone suppression test does not suppress cortisol or AGTH levels. (Choices B, C and E) Adrenal adenoma and carcinoma will have low ACTH levels in combination with the clinical features of Cushing syndrome. The case described has slightly elevated ACTH levels, making adrenal adenoma and adrenal cancer unlikely. Serum ACTH is also low in exogenous glucocorticoid-induced Cushing syndrome.

Educational Objective: Adrenal adenoma and carcinoma will have low levels of ACTH in combination with the clinical features of Gushing syndrome. AGTH levels are elevated in pituitary adenomas, and are suppressed by high-dose, but not low-dose, dexamethasone. Serum ACTH levels are generally markedly elevated in ectopic ACTH production by malignant tumors: even high-dose dexamethasone does not suppress these levels.

203

Q NO 17: A 56-year-old Caucasian female presents to your office with recent weight gain and easy fatigability. Her blood pressure is 160190 mmHg and her heart rate is 80 beats per minute. Her fasting plasma glucose level is 135 mg/dL and her 24-hour urine cortisol excretion is elevated. Further evaluation reveals that her serum cortisol is suppressed by high-dose but not low-dose dexamethasone. Her serum ACTH is borderline elevated. Which of the following is the most likely cause of this patient’s problem?

Page 24: Patho Physio 50q

USMLE WORLD STEP 1 PATHOPHYSIOLOGY

A. Epicardial vessel dilation B. Coronary microvessel dilation C. Capacitance vessel dilation D. Arterial dilation E. Mixed arterial and venous dilation

Explanation:In coronary artery disease, coronary vessel occlusion can be bypassed by the natural existence and compensatory recruitment of coronary collateral vessels to help support blood flow. These collateral microvessels are a network of arterioles that form passageways to major vessels and can supplement blood flow to the myocardium distal to occluded vessels. In the event of myocardial ischemia, collateral microvessels vasodilate and increase collateral blood flow, diverting blood to ischemic areas. This collateral circulation helps to alleviate ischemia and preserve myocardial function. Drugs like adenosine and dipyridamole are selective vasodilators of coronary vessels. Consequently, these agents are often employed in myocardial perfusion imaging studies. In certain conditions, these agents may cause redistribution of blood flow through coronary microvessels or arterioles, possibly reversing collateral blood flow. Vessels within ischemic areas are often maximally dilated and administration of these agents can lead to selective vasodilation of vessels in non-ischemic regions. Decreased pressure and vasodilation of collateral microvessels may then divert blood flow from ischemic areas to non-ischemic areas. This phenomenon, known as coronary steal, decreases blood flow to ischemic areas and may lead to hypoperfusion and potentially worsen existing ischemia. (Choice A) The epicardial vessels refer to the large coronary arteries of the heart and include the right coronary left main, left anterior descending, and circumflex arteries. (Choice C) Capacitance vessels or veins are the main blood vessels that return blood to the heart. They have significant storage capacity and serve as low resistance reservoirs. Veno dilation decreases ventricular volume and allows for a reduction in myocardial oxygen demand secondary to decreased wall tension. Drugs that cause capacitance vessel dilation will have beneficial effects in coronary heart disease. Veno dilation does not normally cause coronary steal. (Choice D) Systemic arterial vasodilation decreases arterial pressure and allows for a reduction in myocardial oxygen demand by decreasing wall tension. (Choice E) Mixed arterial and venous dilation decreases wall tension by reducing arterial pressure and ventricular volume, respectively. The combined effects help to decrease myocardial oxygen demand and are very effective in treating coronary heart disease. Educational Objective: Collateral microvessels are arterioles that form adjacent pathways for blood flow to areas that are distal to occluded vessels. Vasodilators like adenosine and dipyridamole are selective vasodilators of coronary vessels that are often used in myocardial perfusion imaging studies. In coronary steal, blood flow is redistributed from ischemic areas to non-ischemic areas through vasodilate collateral microvessels. Coronary steal can lead to hypoperfusion and worsen ischemia in the occluded artery.

204

Q NO 18: Atherosclerotic lesions of coronary artery limit the potential for increase in blood flow to the myocardium. Some preparations can cause ‘coronary steal phenomenon due to redistribution of blood flow. Which of the following effects of a drug is most likely to be associated with the ‘coronary steal phenomenon’?

Page 25: Patho Physio 50q

USMLE WORLD STEP 1 PATHOPHYSIOLOGY

A. Cellular swelling B. Cell membrane damage C. Glutathione peroxidase production D. Mitochondrial vacuolization E. Nuclear shrinkage

Explanation:Ischemia is characterized by the reduction of blood flow, usually as a result of mechanical obstruction within the arterial system (eg, thrombus). If the flow of blood to the ischemic tissue is restored in a timely manner those cells that were reversibly injured will typically recover. Sometimes however, the cells within the damaged tissue will paradoxically die at an accelerated pace through apoptosis or necrosis after resumption of blood flow. This process is termed reperfusion injury, and is thought to occur secondary to one or more of the following mechanisms: 1) oxygen free radical generation by parenchymal cells endothelial cells, and leukocytes; 2) severe irreversible mitochondrial damage described as “mitochondrial permeability transition”; 3) inflammation which attracts circulating neutrophils that cause additional injury; and 4) activation of the complement pathway. Causing cell injury and further inflammation. When the cells within heart, brain, or skeletal muscle are injured the enzyme creatine kinase leaks across the damaged cell membrane and into circulation (as seen in this patient). (Choice A) Cellular swelling arises secondary to changes in ion concentration and the influx of water. This state is considered a hallmark of reversible injury, and is not directly associated with the leakage of intracellular proteins such as creatine kinase. (Choice C) Glutathione peroxidase actually reduces cellular injury by catalyzing free radical breakdown. The presence of this enzyme is not responsible for the release of creatine kinase. (Choice D) Mitochondrial vacuolization reduces the cellular capacity for ATP generation and is associated with irreversible injury. Creatine kinase release is not directly associated with this mitochondrial change however. (Choice E) Nuclear shrinkage (pyknosis), fragmentation and dissolution characterize irreversible injury of the cell. Creatine kinase release is not directly associated with such nuclear changes however.

Educational Objective: Reperfusion injury is thought to occur secondary to oxygen free radical generation mitochondrial damage and inflammation.

205

Q NO 19: A 65-year-old Caucasian male presents to the ER with sudden onset of right-sided calf and foot pain. His past medical history is significant for hypertension, type II diabetes mellitus, atrial fibrillation and stable angina. Physical examination reveals paleness of the right leg and diminished right popliteal pulse. Immediate angiography is ordered that reveals an obstructive thrombus in the right common femoral artery. The thrombus extraction is followed by a rapid surge of serum creatine kinase level, which is best explained by:

Page 26: Patho Physio 50q

USMLE WORLD STEP 1 PATHOPHYSIOLOGY

A. Sudden increase of left ventricular after load B. Sudden increase in left ventricular filling C. Sudden decrease of left ventricular preload D. Sudden decrease in left ventricular contractility E. Insidious right ventricular hypertrophy

Explanation:Acute atrial fibrillation most likely precipitated the sudden onset of heart failure in this patient. Atrial fibrillation occurs in up to 1O% of patients with severe aortic stenosis (AS). Patients with severe AS may already have a reduced cardiac output. The sudden loss of the contribution of normal atrial contraction to ventricular filling (loss of the atrial systolic kick) decreases left ventricular (LV) preload (end diastolic volume) which can further reduce cardiac output and produce severe hypotension. Additionally, many patients with chronic AS have concentric LV hypertrophy and therefore reduced left ventricular (LV) compliance. Loss of the atrial kick in these patients may mean that a significant increase in mean pulmonary venous pressure is required to maintain the new steady state LV preload. The result may be acute pulmonary edema in addition to hypotension as occurred in this patient. Because of these dangers, cardioversion is indicated for acute atrial fibrillation in patients with severe chronic AS. (Choice A) An acute increase in left ventricular (LV) afterload (mean systolic intraventricular pressure) would be unlikely in a patient with degenerative aortic valve calcification. An increased LV afterload in the setting of reduced mean arterial pressure would have to result from an acute increase in resistance across the aortic valve, whereas in degenerative calcific AS and most other forms of adult AS, the transvalvular obstruction gradually increases over years to decades. (Choices B and D) Since this patient has no evidence of myocardial ischemia on ECGI we may assume that myocardial contractility is roughly unchanged. We may also assume that the degree of aortic stenosis is relatively fixed. Under these circumstances an increase in LV preload would increase net cardiac output (according to the Frank-Starling curve relating preload and stroke volume). Since there is also no reason to suspect any acute change in total peripheral resistance in this patient an increase in cardiac output would increase mean arterial pressure not decrease it as was the case here. In order for there to be a sudden increase in left ventricular (LV) preload (end diastolic volume) there would have to be: a sudden increase in mean left atrial pressure without any change in mitral valve resistance a sudden decrease in mitral valve stenosis an acute increase in LV compliance an acute decrease in LV contractility, and/or a sudden increase in aortic valve regurgitation. (Choice E) As the word insidious implies right ventricular hypertrophy (RVH) develops gradually in response to pulmonic outflow tract obstruction pulmonary hypertension or RV volume overload. RVH is not an acute hemodynamic change rather it is a cardiac structural adaptation to chronic hemodynamic changes. Thus, RVH is rarely responsible for acute symptoms or signs. Educational Objective: In patients with chronic aortic stenosis (AS) and concentric left ventricular hypertrophy:

206

Q NO 20: A 72-year-old Caucasian male who was diagnosed with severe aortic stenosis six months ago presents to the ER with acute pulmonary edema. His blood pressure is 90/60 mmHg and his heart rate is 130 beats per minute with a rhythm that is irregularly irregular. EGG shows atrial fibrillation without significant ST-segment or T-wave changes. Which of the following hemodynamic changes most likely contributed to this patient’s condition?

Page 27: Patho Physio 50q

USMLE WORLD STEP 1 PATHOPHYSIOLOGY

1. the loss of the contribution of atrial contraction to ventricular filling that occurs with acute atrial fibrillation (AF) can reduce left ventricular preload and cardiac output sufficiently to result in dangerous systemic hypotension, and 2. Acute AF might also increase steady state pulmonary venous pressures sufficiently to cause acute pulmonary edema.

207

Page 28: Patho Physio 50q

USMLE WORLD STEP 1 PATHOPHYSIOLOGY

GnRH LH Testosterone A. Decreased Decreased Decreased B. Increased Decreased Decreased C. Increased Increased Decreased D. Increased Increased Increased E. Decreased Increased Decreased

Explanation:“Bitemporal hemianopsia” is a buzzword symptom for a pituitary tumor. Approximately 60% of functional (secreting) pituitary tumors are prolactinomas. Prolactin is a 199 amino acid peptide secreted by the lactotroph cells of the pituitary. Prolactin is responsible for milk production and lactation in postpartum women. The role of prolactin in males is not completely understood. Scientists do know, however, that increased serum levels of prolactin in men or women, from any cause, suppress GnRH. GnRH stimulates the release of LH from the pituitary, which stimulates testosterone production; consequently, suppression of GnRH causes decreased LH and decreased testosterone. Prolactinomas in males generally have a delayed diagnosis for a few reasons; men with prolactinomas do not experience galactorrhea or amenorrhea, and men are often reluctant to report erectile dysfunction. Henceforth, prolactinomas in men are typically much larger at the time of presentation than in women. We know that this particular patient has a large prolactinoma because itis compressing the optic chiasm, as demonstrated by bitemporal hemianopsia. Prolactinomas in females of reproductive age typically cause galactorrhea, amenorrhea, and infertility. Postmenopausal women with prolactinomas are already amenorrheic and infertile, so they present mainly with headaches and visual field defects. (Choices B, C, and D) The choices with increased GnRH levels are incorrect because prolactinomas suppress GnRH production. We can be confident that this patient has a prolactinoma because his serum prolactin level is high, and he has bitemporal hemianopsia. (Choice E) The suppression of GnRH in patients with prolactinomas causes a decrease in LH and testosterone; therefore, Choice E is incorrect.

Educational Objective: Anytime a patient has bitemporal hemianopsia, a pituitary tumor should be suspected. The most common functional pituitary tumor is a prolactinoma. Secreting prolactinomas inhibit the entire axis of GnRH—LH/ESH—sex hormones, causing impotence in men and amenorrhea in women of reproductive age (hypogonadotropic amenorrhea).

208

Q NO 21: A 34-year-old Caucasian male comes to your office with a 3-month history of impotence. Physical examination reveals bitemporal visual Held deficit. Lab results show elevated serum prolactin. Which of the following changes in gonadotropin-releasing hormone (GnRH), luteinizing hormone (LH) and testosterone are most likely responsible for this patient’s symptoms?

Page 29: Patho Physio 50q

USMLE WORLD STEP 1 PATHOPHYSIOLOGY

A.

Glycine B. Glutamate C. Acetylcholine D. Norepinephrine E. Serotonin

Explanation:The use of opioids can lead to the development of tolerance or a decrease in opioid effectiveness and physiological response with continued use. The mechanism for acute opioid tolerance is still uncertain but is postulated to involve phosphorylation of opioid receptors by protein kinase. Chronic tolerance may involve increased adenylyl cyclase activity or nitric oxide levels. In the case of morphine, the neurotransmitter glutamate has also been shown to interact with opioid pathways to modulate morphine tolerance. Glutamate is an excitatory neurotransmitter that binds and activates NMDA receptors. NMDA receptor activation can cause increased phosphorylation of opioid receptors and increased nitric oxide levels which ultimately leads to morphine tolerance. In animal studies, NMDA receptor antagonists, like ketamine, block the actions of glutamate and effectively block morphine tolerance. Additionally, dextromethorphan has also been shown to reverse opioid tolerance through its NMDA antagonistic properties. Thus it appears that glutamate may play a significant role in morphine tolerance. (Choice A) Glycine is a co-agonist for glutamate and is required for the binding of glutamate to NMDA receptors. Binding of both glutamate and glycine is necessary for activation of NMDA receptors. Although glycine is necessary for glutamate binding, it plays no significant role in modulating morphine tolerance. (Choice C) Acetycholine is a neurotransmitter that functions in both the peripheral and central nervous system. It binds to both nicotinic and muscarinic receptors to produce proper nervous system and muscle function. It plays no role in modulating morphine tolerance. (Choice D) Norepinephrine is both a hormone released from the adrenal glands and a neurotransmitter released from noradrenergic neurons. As a hormone, it mainly acts to work on attention and impulsivity. As a neurotransmitter, it mainly functions at postganglionic neurons to activate the sympathetic nervous system. Although norepinephrine dysregulation may have a role in neuropathic pain, it has no role in modulating morphine tolerance. (Choice E) Serotonin is a monoamine neurotransmitter synthesized and released from serotonergic neurons located in the central nervous system and the gastrointestinal system. Although dysregulation of serotonin may play a role in neuropathic pain, it is not involved in modulating morphine tolerance. Educational Objective: Morphine tolerance is a common problem in the treatment of pain. The exact mechanism of tolerance is unknown buy may involve increased phosphorylation of opioid receptors, increased adenylyl cyclase activity, or increased nitric oxide levels. Activation of NMDA receptors by glutamate is believed to enhance morphine tolerance by increasing phosphorylation of opioid receptors and increasing nitric oxide levels. NMDA receptor blockers, like ketamine, block the actions of glutamate and effectively decrease morphine tolerance.

209

Q NO 22: In experimental studies the mechanisms of opioid tolerance are investigated. It is shown that ketamine can block tolerance development to morphine. Which of the following neurotransmitter actions is most likely modulated to achieve the effect described above?

Page 30: Patho Physio 50q

USMLE WORLD STEP 1 PATHOPHYSIOLOGY

A. Phenylalanine-containing food B. High-fat diet C. Smoking D. Iron-containing pills E. Strenuous physical activity F. Direct sunlight exposure

Explanation:Neutrophil elastase is the major protease of extracellular elastin degradation. It is released by neutrophils and macrophages. The major serum inhibitor of extracellular elastase is alpha-antitrypsin (al-AT). Patient B likely has al-AT deficiency, a condition associated with panacinar emphysema and liver cirrhosis. Panacinar emphysema results from the unopposed action of neutrophil elastase on alveolar walls. Smoking dramatically increases the risk of panacinar emphysema in patients with al-AT deficiency. This may be because oxidant products of smoke (including free radicals) can inactivate endogenous al-AT, producing a “functional” αl-AT deficiency as well. Smoking also enhances elastase activity in macrophages and macrophage elastase (unlike neutrophil derived elastase) is not inhibited by al-antitrypsin. (Choice A) A phenylalanine-restricted diet is given to patients with phenylketonuria due to a deficiency of the enzyme phenylalanine hydroxylase. (Choice B) High fat diets have been variably associated with obesity and nonalcoholic fatty liver disease. Dietary fat is not known to affect the development of emphysema or hepatic cirrhosis that may result from al-antitrypsin deficiency. (Choice D) Avoidance of excess dietary iron would be important in conditions associated with systemic iron overload such as hemochromatosis or anemias (e.g. thalassemias) requiring chronic transfusion protocols. (Choice E) Strenuous physical activity would not be contraindicated in such a patient unless the patient had already developed severe panacinar emphysema and/or cirrhosis associated with this antiprotease deficiency. (Choice F) Avoidance of sunlight might be indicated in a patient with a photodermatosis such as cutaneous porphyria or lupus photosensitivity.

Educational Objective: In patients with an al-antitrypsin deficiency, smoking dramatically increases the risk of developing panacinar emphysema.

210

Q NO 23: Serum from patient A seems impairs elastin degradation by neutrophil products in vitro whereas serum from patient B does not have that ability. Patient B should be strongly warned to avoid:

Page 31: Patho Physio 50q

USMLE WORLD STEP 1 PATHOPHYSIOLOGY

A. Kills viruses B. Opsonizes bacteria C. Kills helminths D. Inhibits fungal growth E. Stimulates fibroblasts

Explanation:The cell shown has a bibbed nucleus and is packed with large granules of relatively uniform size. We are told that these granules contain a protein capable of damaging the respiratory epithelium in atopic asthma. The late phase of an atopic asthma attack involves mucosal infiltration by eosinophils, basophils, and neutrophils. Neutrophil proteases could theoretically damage epithelial cells, but neutrophils tend to have multilobed nuclei. Basophil granules contain heparin, histamine, and SRS-A (slow reacting substance of anaphylaxis, a mixture of leukotrienes), which would be unlikely to cause direct damage to epithelial cells. Thus the cell shown is most likely an eosinophil. Eosinophils release major basic protein, a potent anthelminthic toxin that is capable of causing damage to epithelial and endothelial cells. (Choice A) Natural killer cells kill viruses. (Choice B) Major basic protein is an antiparasitic cytotoxin. It is not known to play a role in the opsonization of bacteria. (Choice D) Major basic protein attaches to and disrupts the outer membrane of helminths. It is not known to have an antifungal action. (Choice E) Major basic protein is an antiparasitic cytotoxin not known to directly stimulate fibroplasia or fibrogenesis.

Educational Objective: Major basic protein released by eosinophils normally functions to kill helminths. It is also thought to contribute to the bronchial epithelial damage sustained by patients with atopic (extrinsic allergic) asthma.

211

Q NO 24: A protein isolated from the granules of the cell shown on the slide below is believed to cause damage to the bronchial epithelium in patients with atopic asthma. Which of the following is a known function of this protein?

Page 32: Patho Physio 50q

USMLE WORLD STEP 1 PATHOPHYSIOLOGY

A. Primary polydipsia B. Complete central diabetes insipidus C. Partial central diabetes insipidus D. Nephrogenic diabetes insipidus E. Post obstructive polyuria

Explanation:Vasopressin or antidiuretic hormone (ADH)I is responsible for the maintenance of water balance by regulating water absorption in the kidney. Without ADH, the kidney’s collecting duct cells are impermeable to water causing water to be lost to the body via urine. When ADH is present however water is free to osmotically move across the collecting duct cells. ADH activates G protein-coupled V2 receptors which allow the transposition of aquaporin 2 from their intracellular locations to the luminal cell membrane. At the cell membrane aquaporin lives up to its name by serving as a water channel a “pore” that water passes through. Diabetes insipidus (Dl) is a disease of this water balance system. Patients with Dl pass very watery (dilute) urine making them dehydrated. Because they are dehydrated these patients are also always thirsty. Dl can be partial or complete and is caused by one of two mechanisms—deficiency of ADH called central Dl; or resistance to ADH’s action on the kidneys called nephrogenic Dl. In patients with suspected Dl, a water deprivation is performed. This testis usually done in a hospital setting under close observation. Bodyweight, blood pressure heart rate, urine and plasma osmolality, urine volume, and serum sodium are monitored closely. When two consecutive urine samples show very little change in urine osmolality(<30 mOsm/kg), five units of aqueous vasopressin are given subcutaneously. One hour after injection, the aforementioned values (bodyweight, blood pressure etc.) are measured again. This vasopressin injection differentiates between central and nephrogenic Dl. If following the injection the urine osmolality changes less than 10%, nephrogenic Dl is diagnosed. This scant response to exogenous vasopressin makes perfect sense because patients with nephrogenic Dl already produce enough vasopressin but their kidneys do not respond to the hormone properly. In fact, sometimes levels of vasopressin are drawn in patients with suspected Dl. Normal-to-elevated levels indicate nephrogenic Dl, whereas low levels demonstrate central Dl. Since central Dl is caused by lack of vasopressin one can expect a more robust response to its administration. If urine osmolality increases by 10% or more central Dl is the diagnosis. Furthermore, in patients with complete central Dl, the rise in urine osmolality is typically more than 50%. This particular patient had a robust response indicating that she has complete central Dl. A more moderate response would indicate partial central Dl, which means that some vasopressin is present but not enough to allow normal kidney function.

(Choice A) Primary polydipsia—also called psychogenic polydipsia—is simply excessive consumption of water. The patient’s urine will be dilute (have a low urine

212

Q NO 25: A 35-year-old Caucasian female comes to your office complaining of excessive thirst and frequent urination. Her blood glucose level is 86 mg/dL. You proceed with the standard water deprivation test in this patient. The results of urine osmolality (in mOsm/L) during 4 hours of dehydration are presented below. 1 hour 2 hour 3 hour 4 hour 150 160 160 550 * 5 units of vasopressin administered Which of the following is the most likely diagnosis in this patient?

Page 33: Patho Physio 50q

USMLE WORLD STEP 1 PATHOPHYSIOLOGY

osmolality) because the body is trying to get rid of the excess water. Water deprivation will result in a reliable increase in urine osmolality, and vasopressin administration will not alter test results significantly. Primary polydipsia is a psychological illness; excessive thirst will not have a medical cause. Classically, it is seen in female patients with overt mental illness. Children sometimes present with this disorder as well. (Choice E) Post obstructive polyuria is increased urinary output after surgical (Foley catheter) relief from some sort of urinary obstruction. Suspect it in a patient who has had recently relieved bilateral ureteral or subvesical obstruction. Patients exhibiting this phenomenon will show a normal response to water deprivation.

Educational Objective: More than a 10% increase in urine osmolality following administration of vasopressin during a water deprivation test suggests central Dl. A urine osmolality increase above 50% is strongly suggestive of complete central Dl.

213

Page 34: Patho Physio 50q

USMLE WORLD STEP 1 PATHOPHYSIOLOGY

A. Splitting of S1 that is accentuated on inspiration B. Ejection-type systolic murmur that increases on standing C. Diastolic decrescendo-type murmur that decreases following amyl nitrite inhalation D. Presystolic murmur that disappears with atrial fibrillation E. Splitting of S2 that does not change with respiration Explanation:A patent connection between the right and left atria is a defect that would make a paradoxical embolism possible. Paradoxical emboli originate in the venous system, but cross over into the arterial circulation (bypassing the lungs) via an abnormal connection between the right and left heart. Wide splitting of S2 that does not vary with respiration can result from an atrial septal defect (ASD) a defect that would permit a paradoxical embolism. (Choice A) Assuming the patient had no S4 gallops or aortic ejection click, a split S1 accentuated on inspiration would indicate delayed closure of the tricuspid valve. This could be caused by a right bundle branch block and need not indicate any abnormal connection between the right and left cardiac chambers. (Choice B) A systolic ejection murmur (SEM) generally refers to a mid-systolic crescendo-decrescendo murmur, most commonly the result of aortic stenosis. Hypertrophic obstructive cardiomyopathy may also cause SEM. When in the upright position, venous return to the heart is decreased and the left ventricular end-diastolic volume and stroke volume are reduced, increasing the SEM of hypertrophic obstructive cardiomyopathy. Neither of these lesions by themselves would permit a paradoxical embolus. (Choice C) An early diastolic decrescendo murmur is characteristic of aortic regurgitation (AR). Inhaled amyl nitrite produces marked vasodilatation, resulting in reduction of systemic arterial pressure and decreasing this regurgitant murmur. Isolated AR does not result in an abnormal right-to-left heart connection that would permit paradoxical embolism. (Choice D) “Presystolic accentuation” occurs when the intensity of a diastolic murmur becomes louder just prior to S1 or when a diastolic murmur appears just prior to S1. A presystolic (late diastolic) murmur can result from mitral or tricuspid valve stenosis and/or physiologically increased blood flow across these valves. Presystolic accentuation occurs due to atrial contraction. Atrial fibrillation could eliminate an atrioventricular valve stenotic murmur by removing the atrial contraction during late diastole. However, tricuspid and/or mitral stenosis alone would not permit a paradoxical embolus. Educational Objective: Paradoxical thromboembolism occurs when a blood clot from the venous system crosses directly into the arterial circulation via an abnormal connection between right and left cardiac chambers, such as an ASD or ventricular septal defect. Auscultatory findings in an ASD include a wide and fixed splitting of S2. Additional associations between auscultatory findings and cardiac lesions are as follows: 1. Systolic ejection murmur accentuated by standing: hypertrophic obstructive cardiomyopathy 2. Early diastolic decrescendo murmur decreased by amyl nitrite: aortic regurgitation 3. Late diastolic murmur eliminated by atrial fibrillation: mitral (and/or tricuspid) stenosis

214

Q NO 26: A thrombus originating in the deep veins of the lower extremities is most likely to cause a stroke in a patient with which of the following physical findings?

Page 35: Patho Physio 50q

USMLE WORLD STEP 1 PATHOPHYSIOLOGY

A. Cytomegalovirus B. Babesia divergens C. Toxoplasma gondii D. Isospore belli E. Herpes zoster F. Trypanosoma cruzi D. Cryptococcosis

Explanation:This homeless patient is most likely HI V-infected as Pneumocystis carinii affects exclusively immunocompromised individuals. Now, the patient presents with painful swallowing, which is a characteristic symptom of esophagitis. There are three main causes of HI V-associated esophagitis: Candida, Cytomegalovirus, and Herpes virus. Clinically it is not possible to distinguish which of the three is present as all cause dysphagia (difficulty swallowing) and/or odynophagia (pain on swallowing). Accurate diagnosis, however, is essential for treatment of these patients. Endoscopic and microscopic criteria are given in the table below.

(Choice B) Babesia divergens is transmitted by a tick bite and causes babesiosis. It is endemic in the northeastern United States and manifests with influenza-like symptoms hepatosplenomegaly, and anemia. It often affects asplenic patients. (Choice C) In HI V-positive patients Toxoplasma causes ring-enhancing brain lesions and chorioretinitis. (Choice D) Isospore belli causes profuse, watery diarrhea in HIV patients. It doesn’t play any role in the development of esophagitis. (Choice E) Herpes simplex, not herpes zoster causes esophagitis in HIV patients. (Choice F) Trypanosome cruzi causes Chagas disease (American Trypanosomiasis). Chronic disease leads to cardiomyopathy, achalasia, megacolon, and megaureter. (Choice C) Cryptococcosis causes meningitis in HIV patients, but not esophagitis.

Educational Objective: Infectious esophagitis is common in HI V-positive patients. The most common cause is Candida albicans, although CMV and HSV-1 are also frequently implicated. Diagnosis relies on endoscopic and microscopic findings.

215

Q NO 27: A 32-year-old homeless male presents to the FR with severe chest pain when he swallows food. He has been hospitalized several times recently with pneumocystic pneumonia (PCP). Endoscopic findings include hyperemia and ulcerations of esophageal mucosa. This patient’s condition is most likely caused by which of the following organisms?

Page 36: Patho Physio 50q

USMLE WORLD STEP 1 PATHOPHYSIOLOGY

A. Esophageal cancer B. Gastroesophageal reflux disease C. Duodenal ulcer D. Angina pectoris E. Pulmonary infarction F. Acute cholecystitis

Explanation:Periodic non-peristaltic contractions of the esophagus describe diffuse esophageal spasm (DES). Normally, esophageal muscle contractions are coordinated. Contractions are normally stimulated by esophageal distention by a food bolus; the contractions originate above the site of distention and propel the bolus downwards. In DES, several segments of the esophagus contract at the same time which prevents the propagation of the food bolus towards the stomach. Additionally, these involuntary muscle contractions can be painful. Esophageal manometry studies show disorganized non-peristaltic contractions of the body of esophagus. Simultaneous contractions cause a so-called “corkscrew” esophagus, as seen on barium esophagogram. Symptoms of DES are intermittent dysphagia and occasional chest pain. In typical cases, the chest pain is not associated with exertion and is not relieved by rest. Sometimes, however, the pain may mimic unstable angina in intensity and location. Complete cardiac work-up should be considered in every patient suspected of having DES to rule out a cardiac cause of chest pain. (Choice A) Esophageal cancer causes progressive dysphagia first to solids, later to both solids and liquids. Eventually, patients are unable to eat. (Choice B) The most common symptom of gastroesophageal reflux disease (GERD) is heartburn. This burning, central pain most frequently occurs after a meal or at night. Patients describe DES pain as crampy, not burning. (Choice C) Duodenal ulcers cause epigastric pain that usually occurs 1-3 hours after a meal or at night. Duodenal ulcer pain is relieved by intake of food or antacids, unlike DES pain. (Choice E) Chest pain from a pulmonary embolism is pleuritic (associated with deep breathing) and is often associated with dyspnea, cough, and hemoptysis. (Choice F) Pain from acute cholecystitis is localized to the right upper quadrant of the abdomen. This pain can radiate to the back or the neck. Cholecystitis is often triggered by a large meal and maybe accompanied by fever, nausea, and vomiting.

Educational Objective: Diffuse esophageal spasm (DES) occurs due to uncoordinated contractions of the esophagus. These contractions are both inefficient in propelling food into the stomach and may cause symptoms of dysphagia and chest pain. This chest pain may mimic unstable angina: thus, complete cardiac work-up should be considered in every patient suspected of having DES, so that a cardiac cause may be ruled out.

216

Q NO 28: Esophageal manometric studies performed on a 38-year-old Caucasian male demonstrate periodic non-peristaltic contractions of a large amplitude and long duration. Clinical manifestations of this patient’s condition would most closely mimic which of the following?

Page 37: Patho Physio 50q

USMLE WORLD STEP 1 PATHOPHYSIOLOGY

A. Acute fibrinous pericarditis B. Acute myocardial infarction C. Septic shock D. Constrictive pericarditis E. Cardiac tamponade F. Tension pneumothorax

Explanation:The physical finding of jugular venous distension (JVD) is key here. JVD indicates that there is elevation of the central venous pressure (CVP) in the superior vena cava. The combination of acute-onset CVP elevation (>15cm H2O) with hypotension and tachycardia can occur with cardiac tamponade or tension neumothorax. Because there is no history of chest trauma and no abnormalities on lung auscultation, tension pneumothorax is unlikely. Given the history of an antecedent respiratory illness, the most likely diagnosis is cardiac tamponade due to a serous viral pericarditis and a significant acute pericardial effusion. Observation of Beck’s triad on physical examination — hypotension, distended neck veins, and distant or muffled heart sounds on auscultation — as well as tachycardia, are together indicative of tamponade. A rising CVP in a hypotensive patient signifies that normal compensatory mechanisms are unable to maintain an adequate cardiac output. Low cardiac output and its associated hypoxemia likely explain the patient’s dyspnea and fatigue. The loss of a palpable pulse during inspiration is most likely due to an inspiratory fall in systolic blood pressure. Pulsus paradoxus, or a drop in systolic blood pressure of >10 mmHg on inspiration, is a non-specific sign, but is suggestive of tamponade when coupled with acute onset hypotension, tachycardia, and JVD. (Choice A) Acute fibrinous pericarditis may follow an upper respiratory infection, but would only be expected to cause pleuritic chest pain and a pericardial friction rub. In contrast, the patient in this vignette has signs and symptoms of significant pericardial effusion and cardiac tamponade. (Choice B) A myocardial infarction resulting in a degree of cardiogenic shock could produce the findings of hypotension, tachycardia jugular venous distension, and weak pulses. However, there would likely also be pulmonary edema producing rales on lung auscultation. Moreover, myocardial infarction would be quite unusual in a 34-year-old patient. Finally, ventricular contractile dysfunction and congestive failure would not necessarily cause a significant drop in systolic blood pressure on inspiration (pulsus paradoxus). (Choice C) The initial stage of septic shock is a hyperdynamic circulatory state with a lowered systemic vascular resistance and an increased cardiac output (warm shock). Weak pulses and pulsus paradoxus would not be found. (Choice D) Constrictive pericarditis is a chronic process that requires months to years to produce constriction sufficient to cause tamponade. This patient’s signs and symptoms occurred much more acutely.

217

Q NO 29: A 34-year-old Caucasian male presents to the emergency department with recent-onset dyspnea and fatigue. There is no significant past medical histo except for a mild respiratory illness one week ago. His blood pressure is 80/60 mmHg. His pulse is 120 beats per minute, regular, but weak. The pulse becomes undetectable to palpation during each inspiration. The jugular veins are distended. The lungs are clear to auscultation. Which of the following do you most suspect in this patient?

Page 38: Patho Physio 50q

USMLE WORLD STEP 1 PATHOPHYSIOLOGY

Educational Objective: Cardiac (or pericardial) tamponade presents clinically with hypotension, tachycardia, and an elevated central venous pressure that produces jugular venous distension (JVD). Heart sounds may be muffled on cardiac auscultation and systolic blood pressure may drop more than 10 mmHg on inspiration (pulsus paradoxus).

218

Page 39: Patho Physio 50q

USMLE WORLD STEP 1 PATHOPHYSIOLOGY

A. Diastolic murmur intensity B. Presystolic accentuation of the murmur C. S2-to-opening snap time interval D. Audible S3 E. Audible S4

Explanation:The best auscultatory indicator of the severity of mitral stenosis (MS) is the length of the interval between A2 and the opening snap (OS). The shorter the interval the more severe the stenosis. The OS occurs due to tensing of the mitral valve (MV) leaflets after the valve cusps have completed their opening excursion. The more thickened and fibrotic the MV the earlier this tensing occurs. The A2-OS interval is also inversely correlated with mean diastolic left atrial pressure. (In modern practice the standard for the diagnosis and determination of MS severity is measurement of mean transvalvular pressure gradients via 2-D Doppler echocardiography). (Choice A) Although the intensity of the diastolic rumble of mitral stenosis (MS) may increase as the degree of MS increases this sign is a less reliable indicator of MS severity. This is because the transmission of the flow murmur at a given transvalvular pressure gradient varies among patients depending upon thoracic anatomy. The diastolic rumble is heard best at the cardiac apex (mitral area) with the bell of a stethoscope. (Choice B) There is presystolic accentuation of the murmur of mitral stenosis (MS) because of the increased transvalvular flow associated with left atrial contraction. This accentuation is heard across a range of MS severity. More significant than the degree of presystolic accentuation is its presence or absence. When MS becomes severe enough to precipitate atrial fibrillation, presystolic accentuation of the MS murmur disappears. (Choices D and E) Left-sided SS and/or S4 gallops are generally absent in mitral stenosis (MS), since left ventricular filling is subnormal to normal. When MS is severe enough to produce pulmonary hypertension, patients may develop right-sided heart failure, with dilatation of the right ventricle (RV) and its annulus and possible secondary tricuspid or pulmonic regurgitation. Tricuspid and pulmonic regurgitation might cause a right-sided S3 and/or S4. However these EN gallops would only arise when the maximal degree of MS had been reached and thus are poor indicators of the severity of less serious MS.

Educational Objective: The best, most reliable and most continuous auscultatory indicator of the severity of mitral stenosis is the A2-OS interval. The shorter this interval the more severe the stenosis. Other auscultatory findings that may accompany MS include a diastolic rumble (intensity varies depending on patient anatomy) and pre-systolic accentuation due to left atrial contraction. A right-sided S3 and/or S4 might arise with end-stage MS critical enough to produce severe pulmonary hypertension.

219

Q NO 30: A 45-year-old Caucasian female is hospitalized with exertional dyspnea and fatigue. She recently emigrated from Eastern Europe and has no significant past medical history. Her blood pressure is 110/80 mmHg and her heart rate is 90 beats per minute and regular. You suspect mitral stenosis. Which of the following is the best indicator of the severity of stenosis?

Page 40: Patho Physio 50q

USMLE WORLD STEP 1 PATHOPHYSIOLOGY

Chemistry panel Serum sodium 130 mEq/L Chloride 93 mEq/L Bicarbonate 12 mEq/L Blood urea nitrogen (BUN) 30 mg/dL Serum creatinine 1.2 mg/dL Calcium 10.0 mg/dL Blood glucose 698 mg/dL

Which of the following potassium values would be most likely in this patient?

Intracellular potassium Extracellular potassium A. Decreased increased B. Increased decreased C. Increased increased D. Decreased decreased E. No Change No change

Explanation: This young patient has both clinical and biochemical features of diabetic ketoacidosis (DKA); she has high blood glucose, low bicarbonate, a high anion gap, and decreased sodium. Most patients in DRA have decreased levels of intracellular potassium with normal to increased extracellular potassium levels. Potassium loss occurs via osmotic diuresis induced by glycosuria. Acidosis also pulls potassium from the intracellular compartment leading to normal to elevated serum potassium levels. Lack of insulin is also responsible for movement of potassium outside the cells because insulin normally promotes the intracellular movement of potassium. The net result of all these events is low total body potassium and low intracellular potassium—with normal to increased extracellular potassium. Then when insulin and intravenous fluids are given to resuscitate the dehydrated and hyperglycemia patient, they push the potassium back into the cells, which cause a precipitous drop in serum potassium because there is still an overall potassium deficiency. When a patient comes to the hospital in DKAI that patient will have low total potassium, even though labs will return with a normal or even elevated serum potassium level. (Choice B) Increase in intracellular potassium with decreased extracellular potassium is seen with glucose and insulin administration. Stimulation of beta-adrenergic receptors also causes an increased transfer of potassium to the intracellular compartment. (Choice C) The renal excretion of potassium is impaired in most patients with chronic renal failure, resulting in high intra- as well as extracellular potassium. (Choice D) A decrease in both intra- and extravascular potassium is seen with mineralocorticoids excess, diuretic use, and gastrointestinal losses.

Educational Objective: Most patients with diabetic ketoacidosis have normal to increased serum potassium levels despite low intracellular potassium. Replacement of potassium is a crucial step in management of patients with diabetic ketoacidosis.

220

Q NO 31: A 20-year-old female presents to the ER with extreme weakness, abdominal pain, and nausea. For the past few weeks, she has had polyuria and excessive thirst. She unintentionally lost 10 lbs in last month. Physical examination shows tachycardia and dry mucus membranes. Laboratory studies show:

Page 41: Patho Physio 50q

USMLE WORLD STEP 1 PATHOPHYSIOLOGY

A. Decrease in cardiac output B. Increase in sympathetic tone C. Decrease in lung capacity D. Renal artery atherosclerosis E. Aortic stiffening

Explanation:This patient’s systolic blood pressure (SBP) exceeds the 140/90 mmHg cutoff for the diagnosis of hypertension (and initiation of antihypertensive therapy). However, the patient’s diastolic blood pressure (DBP) is within a normal range. After age 50, the pattern of isolated systolic hypertension (ISH) becomes quite common. SBP greater than 160 mm Hg with a DBP below 90 mm Hg is found in approximately 20% to 30% of all people 80 and older. ISH is caused by age-related decreases in the compliance of the aorta and its proximal major branches. Numerous alterations in vessel wall structure and function, including atherosclerotic changes, have been proposed to explain this stiffening. (Choice A) A primary decrease in cardiac output (CO) tends to decrease mean arterial pressure at any given value of total peripheral resistance (TPR), as MAP=CO x TPR. In contrast this patient’s MAP = DBP + (1/3) (SBP—DBP) = 106mm Hg1 which is elevated above normal adult values of 90—100mm Hg. (Choice B) Although sympathetic cardio vascular tone does generally increase with age this tends to increase TPR more than it affects aortic compliance. Increased sympathetic vasoconstriction in muscular arteries and arterioles (the vessels contributing to the majority of systemic vascular resistance) would tend to elevate both SBP and DBP. (Choice C) A decrease in (total) lung capacity might be associated with some increase in pulmon avascular resistance. However, this would not necessarily have a significant effect on arterial pressures in the systemic circulation. (Choice D) Renal artery atherosclerosis and renal artery stenosis in particular, can result in renovascular hypertension due to excess activation of the renin-angiotensin-aldosterone system. Any resultant hypervolemia or increased total peripheral (systemic) vascular resistance would tend to elevate both SBP and DBP.

Educational Objective: An age-related decrease in compliance (increased stiffness) of the aorta and its proximal major branches often causes isolated systolic hypertension (ISH).

221

Q NO 32: A 78-year-old Caucasian male presents bra routine check-up. His blood pressure is 180/70mm Hg and his heart rate is 75 beats per minute. Physical examination findings are within normal limits. You explain to the patient that his hypertension is most likely caused by age-related:

Page 42: Patho Physio 50q

USMLE WORLD STEP 1 PATHOPHYSIOLOGY

A. AB. BC. CD. DE. EF. F

Explanation:The hemodynamic profile shows an abnormal pressure gradient between the left ventricle (LV) and the aorta (Ac) during systole (see arrows in graph below), indicating significant aortic stenosis (AS).

The intensity of the murmur of AS is directly related to the magnitude of the LV-to-aorta pressure gradient. Thus the murmur in this patient would be loudest at point B and less intense at point A. (Choices A and C) The murmur of AS is a systolic ejection-type crescendo-decrescendo murmur that starts after the first heart sound, following the opening of the aortic valve (time A). It typically ends before the A2 component of the second heart sound (time C). At times A and C, the left ventricular and aortic pressures are nearly equal so that a murmur due to flow across the aortic valve would be unlikely at these points. (Choices D and E) These points occur during diastole, when the aortic valve is closed and the mitral valve is open. There is no abnormally elevated pressure gradient between the left atrium and left ventricle, consistent with normal unobstructed diastolic filling of the left ventricle. Since turbulent flow due to a high pressure

222

Q NO 33: A 73-year-old Caucasian male presents to your office following repeated episodes of exertional dyspnea. Physical examination reveals a cardiac murmur. The patient is referred to cardiologist or further evaluation. Cardiac catheterization is performed and the findings are shown on the slide below. Which of the time points indicated on the slide best corresponds to the peak of the murmur intensity in this patient?

Page 43: Patho Physio 50q

USMLE WORLD STEP 1 PATHOPHYSIOLOGY

gradient is generally required to produce a murmur the patient would not have a murmur at times D or E. (Choice F) This time point corresponds to atrial contraction just prior to ventricular systole. There is no abnormally elevated pressure gradient between the left atrium and left ventricle consistent with normal unobstructed filling of the left ventricle.

Educational Objective: Aortic stenosis (AS) may cause exertional syncope. The murmur of AS is a systolic ejection-type, crescendo- decrescendo murmur that starts after the first heart sound and typically ends before the A2 component of the second heart sound. The intensity of the AS murmur is proportional to the magnitude of the left ventricle-to-aorta pressure gradient during systole.

223

Page 44: Patho Physio 50q

USMLE WORLD STEP 1 PATHOPHYSIOLOGY

A. Holosystolic murmur intensity B. Presystolic component of the murmur C. S2 to opening snap (OS) time interval D. Presence of audible S3 E. Presence of audible S4

Explanation:Under most modern clinical circumstances, the anatomy and severity of mitral regurgitation (MR) are best delineated by2D and Doppler echocardiography. Among auscultator findings, the best indicator of a high regurgitant volume indicating severe MR with left ventricular volume overload is the presence of a left ventricular S3 gallop. A left ventricular (LV) S3 gallop reflects an increased rate of filling of the LV during mid diastole. It can be heard as a consequence of MR and in this condition reflects the relatively high volume of regurgitant flow which is “recycled” back into the LV during diastole. (Choice A) If the volume of left ventricular blood pumped backed into the left atrium during systole, or regurgitant volume, is used as a measure of severity of mitral regurgitation (MR), then one might expect the murmur of MR to become louder as regurgitant volume increased. However, this would only apply to an anatomically fixed effective regurgitant orifice (ERO). In the clinical setting, patients with higher regurgitant volumes may also have larger EROs, such that systolic transvalvular flow resistance and the degree of turbulence in the regurgitant jet (which accounts for the intensity of the murmur of MR) may not be strongly correlated with regurgitant volume. (Choice B) The murmur of mitral regurgitation is either holosystolic or, in some cases of mitral valve prolapse, midsystolic. In some cases of severe MR, a diastolic rumble produced by a high rate of flow across a normal sized diastolic mitral orifice may be heard. The latter amounts to a functional murmur (relative mitral stenosis) but is a less reliable finding with a high regurgitant volume than is the presence of an S3. (Choice C) The S2 to opening snap interval is a diastolic interval between the second heart sound (specifically A2) and the tensing of a stenotic mitral valve. It is a parameter of mitral stenosis, not mitral regurgitation. (Choice E) In a patient with mitral regurgitation, a left ventricular S4 gallop would most likely be a sign of left heart failure, indicating LV dilatation and the reaching of the limit of LV compliance during end diastole. However, many patients with severe MR have not yet developed left sided heart failure. In the latter group of patients, a left sided S3 would be a more likely finding on cardiac auscultation than a left sided S4.

Educational Objective: In a patient with mitral regurgitation (MR), the most reliable auscultator finding indicating a high regurgitant volume (severe MR) and left ventricular volume overload is a left sided S3 gallop intensity of a holosystolic murmur due to MR may not correlate well with regurgitant volume. A left sided S4 would suggest end stage decompensation of severe MR to left ventricular failure; however, many patents with severe MR may not have developed left heart failure.

224

Q NO 34: A 52-year-old Caucasian male presents to your office for a routine check-up. He says that ‘some cardiac problems were detected during his previous visit to the doctor. Physical examination reveals a holosystolic murmur at the apex that radiates to the axilla. Which of the following is the best indicator of the severity of this patient’s problem?

Page 45: Patho Physio 50q

USMLE WORLD STEP 1 PATHOPHYSIOLOGY

A. Increased plasma oncotic pressure B. Increased interstitial fluid pressure C. Decreased capillary permeability D. Increased tissue lymphatic drainage E. Decreased circulating aldosterone level

Explanation:Progressive exertional dyspnea in a heavy smoker suggests chronic bronchitis/emphysema (COPD). The right ventricular dilatation and elevated central venous pressure indicate secondary cor pulmonale. A high central venous pressure raises the hydrostatic pressure in capillary bed venules, thereby increasing net plasma filtration, especially in dependent tissues. If there is a compensator increase in tissue lymphatic drainage to counteract the interstitial fluid increase, edema does not develop. Only when the venous pressure and net capillary filtration have risen sufficiently to overwhelm the resorptive capacity of tissue lymphatics does edema appear. (Choice A) While an increase in plasma oncotic pressure would oppose edema formation, this does not occur in COPD. (Choice B) The interstitial fluid pressure is increased in edema. (Choice C) A decrease in capillary permeability (filtration coefficient) would oppose edema formation, but patients with chronic bronchitis are not known to have decreased capillary permeability. (Choice E) A decrease in circulating aldosterone could decrease intravascular volumes and pressures, counteracting edema formation. However, patients with cor pulmonale tend to have elevated levels of aldosterone in response to the low cardiac output.

Educational Objective: When the central venous pressure (CVP) is increased, as in right heart failure, the interstitial fluid pressure rises due to an increase in net plasma filtration. As the interstitial fluid pressure increases, so does lymphatic drainage. Increased lymphatic drainage can compensate for moderate CVP elevations to prevent the development of clinically apparent interstitial edema. With large CVP elevations, the net capillary filtration increases in excess of the lymphatic reabsorptive capacity and overt edema develops.

225

Q NO 35: A 53-year-old male with a heavy smoking history suffers from progressive exertional dyspnea and wheezing. Echocardiogram shows moderate dilatation of the right ventricle and increased central venous pressure. No lower extremity edema is observed on physical examination. The absence of edema is best explained by which of the following compensatory mechanisms?

Page 46: Patho Physio 50q

USMLE WORLD STEP 1 PATHOPHYSIOLOGY

A. Severe fatigue and exhaustion B. Atrial fibrillation C. Acute pulmonary edema D. Lower extremity swelling E. Arterial embolism

Explanation:The patient with curve 2 has reduced left atrial compliance (dV/dP). As a result, severe mitral regurgitation results in a large increase in the new steady state left atrial, pulmonary venous, and pulmonary arterial pressures. This picture characterizes acute mitral regurgitation (MR) that may be due to spontaneous rupture of chordae tendineae, infective endocarditis with destruction of valve leaflets or chordal rupture ischemia or rupture of a papillary muscle or failure of a prosthetic valve. Such acute severe MR does not provide enough time for left atrial adaptation to the regurgitant volume overload. Acute pulmonary edema therefore is likely to result. Patients with severe MR (characterized by a high ratio of regurgitant to forward left ventricular stroke volume) may be hemodynamically classified along a spectrum of associated left atrial compliance. Severe acute MR is generally accompanied by a normal left atrial (LA) compliance. More chronically developing MR1 as could result from myxomatous degeneration or mitral valve prolapse permits adaptive dilatation of the LA and thinning of its wall accompanied by an increase in LA compliance— corresponding to curve 1. In the latter case, a given regurgitant volume from the left ventricle results in a lower steady state elevation in LA pressures. There is also a lesser elevation in pulmonary vascular pressures than occurs with normal LA compliance in acute MR. (Choice A) Fatigue and exhaustion are symptoms of a low cardiac output state. The latter may be found in any patient with severe MR. Thus, patients with severe MR can experience these symptoms whether they have a low or a high left atrial compliance. (Choices B and E) Atrial fibrillation would be more likely to occur in a patient with curve corresponding to chronic severe mitral regurgitation. As explained above, the latter permits left atrial dilatation, which, in addition to increasing age, increases the probability of atrial fibrillation. Atrial mural thrombosis and thromboembolism are also more likely with left atrial dilatation. (Choice D) Lower extremity swelling is a sign of right sided heart failure. Right ventricular (RV) failure can occur either in acute mitral regurgitation with normal left atrial (LA) compliance (curve 2) and marked pulmonary hypertension or in decompensation of chronic mitral regurgitation with increased LA compliance (curve 1).

226

Q NO 36: Two left atrial (LA) compliance curves in patients with mitral regurgitation are given in the diagram below. Compared to the patient having LA compliance shown by curve the patient with the curve 2 reading is most likely to suffer:

Page 47: Patho Physio 50q

USMLE WORLD STEP 1 PATHOPHYSIOLOGY

Educational Objective: Patients with severe acute mitral regurgitation (MR) who have a near normal left atrial (LA) compliance tend to develop marked pulmonary hypertension and pulmonary edema. Patients with severe mitral regurgitation that develops chronically acquire an increase in LA compliance. They are therefore less prone to pulmonary hypertension/edema but are more prone to atrial enlargement, fibrillation, and mural thromboembolism. Any patient with severe MR can develop fatigue and also eventual signs of right sided heart failure.

227

Page 48: Patho Physio 50q

USMLE WORLD STEP 1 PATHOPHYSIOLOGY

A.

Cortisol B. Thyroxine C. Vasopressin D. Prolactin E. Somatomedin C

Explanation:Unlike secretion of other pituita hormones prolactin is under tonic (constant) inhibition by dopamine secretion from the hypothalamus. Hypothalamic destruction causes hyperprolactinemia by loss of this tonic inhibition. Dopamine inhibits prolactin production by acting on the D2 dopamine receptor of lactotrophs (the prolactin-producing cells of the pituitary). Medications such as phenothiazines also act on this receptor and cause hyperprolactinemia at certain doses. On the other hand there are several putative prolactin-releasing factors that do not yet have clear physiological roles. Some of the hormones that may increase prolactin levels are thyrotrophin-releasing hormone vasoactive intestinal peptide, oxytocin, and vasopressin. (Choices A, B, and E) in contrast to prolactin, other pituitary hormones (growth hormone thyroid-stimulating hormone adrenocorticotropic hormone luteinizing hormone and follicle-stimulating hormone) are inhibited by hypothalamic lesions because the dominant effect of the hypothalamus on these hormones is stimulatory. Thus hypothalamic lesions will ultimately decrease cortisol, thyroxine, and insulin-like growth factor (somatomedin C). (Choice C) Vasopressin is produced in the supraoptic and paraventricular nuclei of the hypothalamus and transported to the posterior pituitary for release. Lesions of the hypothalamus decrease vasopressin levels.

Educational Objective: The dominant effect of the hypothalamus on prolactin secretion is inhibitory via dopamine production. Prolactin regulation by dopamine is a commonly tested concept on USMLE Step 1.

228

Q NO 37: A group of investigators is studying central nervous system control mechanisms of hormone balance. During an experiment, they destroy a significant portion of the hypothalamus in a lab animal. Which of the following substances is most likely to be elevated as a result of this experiment?

Page 49: Patho Physio 50q

USMLE WORLD STEP 1 PATHOPHYSIOLOGY

Chemistry panel Serum sodium 120 mEq/L Serum potassium 4.0 mEq/L Blood urea nitrogen (BUN) 20 mg/dL Serum creatinine 0.8 mg/dL Calcium 9.0 mg/dL Blood glucose 98 mg/dL Urine osmolality 360 mOsmol/kg

Which of the following additional finding is most likely to be present in this patient?

A. Peripheral edema B. Bilateral crackles at the bases C. Increased B-type natriuretic peptide D. Dilute urine E. Normal total body volume

Explanation:This patient most likely has the syndrome of inappropriate antidiuretic hormone (SIADH). Small cell carcinomas of the lung, formerly called “oat cell carcinomas” consist of cells that contain dense neurosecretory granules and can produce hormones like ADH. The production of ADH is constitutive, meaning that the lung tumor pumps out ADH regardless of feedback. Especially because there is a strong correlation between smoking and small cell carcinomas, we can deduce that this man’s lung mass is most likely one such tumor. It is common for patients with small cell carcinomas to present with symptoms related to the hormones produced by the tumor. In this case, the patient’s low sodium level caused his altered mental status. Low sodium levels can even cause seizures. Initially in SIADHI the extra ADH leads to excessive water absorption from the kidneys, causing a mild hypervolemia early in the disease. The excess body water suppresses the renin-aldosterone axis, causing low aldosterone. As aldosterone is responsible for sodium absorption from the distal tubule, low aldosterone levels lead to natriuresis (salt- wasting via the urine). Eventually, most patients with SIADH will equilibrate with a near-normal total body volume and low plasma osmolality (because of the salt-wasting). Normal ADH secretion by the hypothalamus is regulated by changes in plasma osmolality and intravascular volume. In this patient, the ADH levels is elevated despite low plasma osmolality because the secretion of ADH is not regulated by changes in osmolality; it is produced constitutively. Characteristically, patients with SIADH do not have gross hypervolemia, nor do they have peripheral edema. (Choices A, B, and C) Pedal edema bibasilar crackles and increased serum B-type natriuretic peptide (BNP) are the features of congestive heart failure volume overload. Although patients with SIADH have mild hypervolemia early in the disease these patients do not experience the degree of volume overload that would present with the above symptoms. (Choice D) The patient cannot have dilute urine because we are told the urine osmolality (360 mOsmol/kg): memory or a lab reference will reveal that this patient’s urine osmolality is high. Dilute urine with low plasma osmolality indicates primary polydipsia, an excess of water consumption. Characteristic features of SIADH are a near-normal intravascular volume high urinary osmolality, high urinary sodium and low plasma osmolality.

229

Q NO 38: A 60-year-old smoker is brought to the hospital because of altered mental status. Chest x-ray showed a mass in the lung. Labs are as follows:

Page 50: Patho Physio 50q

USMLE WORLD STEP 1 PATHOPHYSIOLOGY

Educational Objective: SIADH is characterized by concentrated urine low plasma osmolality, increased urinary sodium and relatively normal body volume. Hypothyroidism and hypercortisolism should be ruled out before the diagnosis of SIADH is made in real practice although the lung mass described makes SIADH the most likely disease present in this patient.

230

Page 51: Patho Physio 50q

USMLE WORLD STEP 1 PATHOPHYSIOLOGY

PTH Calcitonin 1, 25-dihydrocholecalciferol A. Increase Decrease IncreaseB. Increase Increase DecreaseC. Decrease Decrease DecreaseD. Decrease Increase DecreaseE. Increase Decrease Decrease

231

Q NO 39: A group of investigators is studying calcium homeostasis. They administer calcium gluconate by continuous IV infusion and take blood samples. Which of the following changes from baseline values would occur?

Page 52: Patho Physio 50q

USMLE WORLD STEP 1 PATHOPHYSIOLOGY

Explanation:Serum calcium is tightly regulated: calcium enters the bloodstream from the bones and from intestinal absorption and calcium is excreted in the urine. This balance is mainly regulated by circulating PTH and 1, 25-dihydrox vitamin D. PTH battles low serum calcium levels by three mechanisms: increasing bone resorption, which releases calcium from the bone into the circulation: increasing the gastrointestinal absorption of calcium by creating the active form of calcium-absorbing vitamin D, and increasing the renal absorption of calcium. PTH release is inversely related to serum calcium concentrations in a negative feedback loop. Calcitonin is secreted by parafollicular C cells in the thyroid and is mainly regulated by serum calcium. In contrast to PTH calcitonin provides negative feedback for calcium concentrations. That is calcitonin battles high serum calcium by decreasing bone absorption via osteoclast inhibition. In normal physiology, however calcitonin is not a dominant hormone. For instance, total thyroidectomy, which causes virtually negligible levels of calcitonin, does not cause significant calcium level alterations. Infusion of intravenous calcium directly raises serum calcium. As calcium and PTH are inversely related PTH levels will decline; hence, the formation of 1, 25-dihydroxyvitamin D will diminish. The increase in calcium levels will prompt the release of calcitonin, a calcium antagonist, as the body tries to reestablish homeostasis. (Choice A) These are the values expected in hypocalcemia. A decrease in serum calcium will increase PTH release and decrease calcitonin release. Elevated PTH levels will increase the formation of 1 25 dihydroxy vitamin D. (Choice B) These are the values one would expect from chronic renal disease, a condition often accompanied by “secondary hyperparathyroidism.” Chronic renal failure decreases the activity of 1-alpha hydroxylase causing the formation of 1, 25-dihydrox vitamin D to be decreased, ultimately resulting in hypocalcemia. PTH release is encouraged both by these low calcium levels and by loss of negative inhibition from 1, 25 dihydroxy vitamin D. Despite the relative hypocalcemia of renal failure, levels of calcitonin are increased because the excretion of calcitonin is impaired. (Choice C) Low levels of circulating PTH in hypoparathyroidism result in the decreased formation of 1, 25-hydroxy vitamin D. The ensuing hypocalcemia causes serum calcitonin to decrease as well.

Educational Objective: In response to calcium loading, PTH decreases, calcitonin increases, and the renal synthesis of the active form of vitamin D decreases.

232

Page 53: Patho Physio 50q

USMLE WORLD STEP 1 PATHOPHYSIOLOGY

UE: upper esophagus ME: middle esophagus LES: lower esophageal sphincter

Which of the following is the most likely diagnosis in this patient?

A. Cricopharyngeal dysfunction B. Diffuse esophageal spasm C. Achalasia D. Gastroesophageal reflux E. Mallory-Weiss syndrome

Explanation:After the contents of the oral cavity are collected on the tongue and propelled into the pharynx, involuntary contraction of the pharyngeal muscles pushes the material into the esophagus. The cricopharyngeus muscle contracts behind the food bolus initiating a peristaltic wave to propel the bolus downward; this contraction is shown on the first diagram as an upward deflection on both the normal and patient graphs. The second diagram shows normal peristalsis of the middle esophagus. Under normal conditions when the food bolus reaches the lower esophagus, the lower esophageal sphincter relaxes to allow the material to enter the stomach. The third diagram portrays the decrease in LES pressure that corresponds to its relaxation in the normal patient. Esophageal manometry of the patient A reveals normal contraction of upper esophageal sphincter, decreased peristalsis of the midesophagus illustrated by a lower amplitude of pressure increase on the second diagram, and no LES relaxation. These manometric findings are consistent with a diagnosis of achalasia (Choice C). This motor dysfunction is associated with an elevated resting pressure of the LES and inability of the LES to relax during swallowing. (Choice A) Cricopharyngeal dysfunction results from a failure of the cricopharyngeus to relax during swallowing and causes a choking or “food sticking” sensation and pain with swallowing. A video fluoroscopic swallowing study is used for diagnosis. (Choice B) Diffuse esophageal spasm (DES) is an esophageal motility disorder that presents with severe, non- cardiac chest pain. It occurs due to uncoordinated contractions of the esophageal body. Esophageal manometry shows multiple contractions on tracings from middle and lower esophagus.

233

Q NO 40: A 43-year-old male (Patient A) is being evaluated for an esophageal disorder. Esophageal manometry tracings after a single swallow of 5 ml of water are shown on the slide below.

Page 54: Patho Physio 50q

USMLE WORLD STEP 1 PATHOPHYSIOLOGY

(Choice D) Reflux of gastric contents into the distal esophagus occurs due to low tone of lower esophageal sphincter and its failure to contract after the transfer of food to stomach. Esophageal manometry in patients with GERD would show decreased LES pressure. (Choice E)Tears of esophageal mucosa at the esophagogastric junction are called Mallory-Weiss tears. These tears are caused by forceful retching and vomiting and are common in alcoholics. Mallory-Weiss tear is nota motility disorder and is not associated with abnormalities of esophageal manometry.

Educational Objective: Decreased esophageal body peristalsis and poor relaxation of the LES on manometry are typical for achalasia. Achalasia presents with progressive dysphagia, chest pain, food regurgitation and aspiration. Barium swallow shows a dilated esophagus and a “bird’s beak” deformity of the LES.

234

Page 55: Patho Physio 50q

USMLE WORLD STEP 1 PATHOPHYSIOLOGY

A. Oral

glucose tolerance test B. 24-hour urinary cortisol excretion C. Serum prolactin level D. Serum TSH level E. Electrocardiogram (ERG)

Explanation:The clinical features described above are consistent with hypothyroidism. Thyroid hormones are responsible for the physiologic function of most organ systems; therefore, dysfunction of the thyroid gland (hypothyroidism or hyperthyroidism) leads to widespread organ system manifestations. The skeletal system is involved in both hyper and hypo-thyroidism. Hypothyroid patients typically complain of the fatigability described in this patient, and also of muscle pain, and cramping. Biopsy of these muscles usually shows pale muscle fibers with decreased striation and deposition of a mucinous material. There is atrophy of the type II muscle fibers. As with any myopathic process, creatinine phosphokinase (CPK) levels can be increased in patients with hypothyroidism. (Choice A) Oral glucose tolerance test is used to diagnose diabetes mellitus. This patient does not have any features suggestive of diabetes mellitus; i.e. he has no polyuria, polydipsia, or weight loss. Furthermore the oral glucose tolerance test has been replaced by fasting blood glucose levels in screening most patients for diabetes mellitus; the oral glucose tolerance test is reserved only for a selected patient populations. (Choice B) The patient described above does not have features of Cushing syndrome. Symptoms of Cushing syndrome include central obesity, abdominal skin striae, proximal muscle weakness hypertension and increased supraclavicular deposition of fat. The 24-hour urine cortisol excretion is a screening test for Cushing syndrome and is not indicated when a thyroid disorder is suspected. (Choice C) Hyperprolactinemia presents with hypogonadism in males. In typical cases of hyperprolactinemia, there is no involvement of muscle system. Although it is true that serum prolactin levels can be elevated in patients with hypothyroidism, establishing this patient’s thyroid status is the best next step in evaluating his fatigability. (Choice E) CPK exists in three isoforms: MM, MB and BB. CPK MB is confined to the cardiac muscle; BB is present in the nervous system and MM is present in the skeletal muscle. Damage to the respective organ system will elevate the level of that specific CPK isoform. If this patient did have a myocardial infarction the CPK would increase and it would be the MB isoform. However the patient has no symptoms suggestive of ischemic cardiac disease; and the myopathy of hypothyroidism accounts for the increase in CPK (MM isoform). Educational Objective: Hypothyroidism is a common cause of elevated CPK level because of hypothyroid myopathy. Sometimes it can be the first manifestation of hypothyroidism. The other common causes of elevated CPK include medications such as HMG Co-A reductase inhibitors (statins), autoimmune disease (polymyositis / dermatomyositis) and muscular dystrophies (like Duchenne muscular dystrophy).

235

Q NO 41: A 45-year-old Caucasian male undergoes evaluation for intense fatigability. He denies any chest pain dyspnea, palpitations or lower extremity edema. He has no other medical problems. His laboratory values are significant for elevated creatinine phosphokinase (CPK) level. Which of the following is the most reasonable next step in managing this patient?

Page 56: Patho Physio 50q

USMLE WORLD STEP 1 PATHOPHYSIOLOGY

A. 17-hydroxylase B. 21-hydroxylase C. 11-hydroxylase D. Desmolase E. 5-alpha-reductase

Explanation:Congenital adrenal hyperplasia (CAH) is a group of disorders that result from defects in the enzymes of cortisol biosynthesis in the adrenal gland. 21-hydroxylase deficiency is the most common type of CAH, accounting for 90% of patients. The enzyme 21-hydraylase catalyzes the conversion of 17-hydroxy-progesterone to 11-deoxycortisol in the zona fasciculata, and the conversion of progesterone to deoxycorticosterone in the zona glomerulosa. When there is a deficiency of 21-hydror’lase, the adrenal gland cannot synthesize cortisol efficiently. Low cortisol

236

Q NO 42: A newborn female is hypotensive and hypoactive. She also has labial fusion and clitoromegaly. The prenatal period and delivery were uncomplicated. Laboratory investigation reveals increased urinary 17-hydroxyprogesterone excretion and decreased 11-deoxycortisone excretion. Which of the following enzymes is most likely to be deficient in this patient?

Page 57: Patho Physio 50q

USMLE WORLD STEP 1 PATHOPHYSIOLOGY

levels stimulate ACTH release from the pituitary gland. In this setting, elevated ACTH levels serve to increase the production of adrenal androgens, because the accumulating high levels of cortisol precursors are diverted towards the adrenal androgen biosynthetic pathway (see diagram). The patient described in the vignette has clinical features of cortisol and aldosterone deficiency (hypotension), and androgen excess (labial fusion and clitoromegaly). (Choice A) 17-hydroxylase deficiency accounts for less than 1% of patients with CAH. 17-hydroxylase converts pregnenolone to 17-hydroxypregnenolone and progesterone to 17-hydroxyprogesterone. This enzymatic pathway is active in both the adrenal gland and the gonads. ACTH levels are high as a result of decreased cortisol production, thus stimulating the formation of deoxycorticosterone and corticosterone in the adrenals. The result is low renin levels, hypertension, and hypokalemia. In females, the genitalia are normal at birth. However, these females have delayed puberty (no production of sex steroid) and are hypertensive (excessive production of deoxycorticosterone and corticosterone). Males are under virilized and hypertensive. (Choice C) 11-β-hydroxylase converts deoxycorticosterone to corticosterone and 1-deoxycortisol to cortisol. Like 21-hydroxylase deficiency, this enzymatic deficiency presents with ambiguous genitalia and increased androgen levels at birth. Because the production of deoxycorticosterone (potent mineralocorticoid) is increased, these patients have features of mineralocorticoid excess (in contrast to the symptoms of mineralocorticoid deficiency that occur with 21-hydroxylase deficiency). (Choice D)20, 22-desmolase converts cholesterol to pregnenolone. Because this is the first enzyme in the steroidogenic pathway, the formation of all steroid hormones is affected. Cholesterol and cholesterol esters accumulate in the adrenal glands. This type of CAH is extremely uncommon and usually carries a bad prognosis. (Choice E) 5-a-reductase deficiency causes ambiguous genitalia in males, as a result of a defective conversion of testosterone to dihydrotestosterone. This is not a form of CAH because cortisol production is not affected. Dihydrotestosterone is responsible for the fusion of the labial folds during normal male fetal development. Low dihydrotestosterone levels during fetal life leads to poor labial fusion in males. Depending on the severity of the defect the external genitalia in males vary from pseudovaginal perineoscrotal hypospadias to labial fusion with microphallus. At puberty, affected males show signs of virilization, such as increased muscle mass, deepened voice and phallus enlargement, because these pubertal features are dependent on testosterone. After puberty, males have a high testosterone level, with low dihydrotestosterone levels and mildly elevated FSH. Females with 5-α-reductase deficiency do not have ambiguous genitalia.

Educational Objective: 21-hydroxylase deficiency is the most common cause of CAH. It may result in mineralocorticoid deficiency and is accompanied by increased 17-hydroxyprogesterone secretion.

237

Page 58: Patho Physio 50q

USMLE WORLD STEP 1 PATHOPHYSIOLOGY

Liver studies Albumin 4.2 mg/dLTotal protein, serum 6.8 mg/dLTotal bilirubin 2.8 mg/dLDirect bilirubin 0.8 mg/dLAlkaline phosphatase 90 U/LAspartate aminotransferase (SGOT) 26 U/LAlanine aminotransferase (SGPT) 32 U/L

This patient most likely suffers from:

A. Gilbert syndrome B. Dubin-Johnson syndrome C. Acute viral hepatitis B D. Acute viral hepatitis C E. Acute alcoholic hepatitis F. Wilson’s disease

Explanation:The hepatic metabolism of bilirubin occurs in the following four stages: uptake from the bloodstream; storage within the hepatocyte; conjugation with glucuronic acid; and biliary excretion. In the normal individual, serum total bilirubin is 0.2-1 mg/dL, of which <0.2 mg/dL is the direct fraction. Typically, elevated conjugated bilirubin levels are suggestive of hepatobiliary disease (eg cirrhosis or hepatitis) because the bilirubin conjugates will reflux back into the plasma when the secretion of conjugated bilirubin into the bile is slowed. In contrast, elevated unconjugated bilirubin levels typically indicate increased bilirubin formation (such as that seen in hemolysis) or a slowing in bilirubin conjugation (such as that seen in this patient, who has Gilbert syndrome). Gilbert syndrome is a common familial disorder of bilirubin glucuronidation in which the production of UDP glucuronyl transferases (enzymes that mediate glucuronidation of various substances) is reduced. Approximately 9% of individuals in Western countries are homozygous for this mutation with another 30% heterozygous and asymptomatic. The diagnosis is suggested in those patients with no apparent liver disease who have mild unconjugated hyperbilirubinemia thought to be provoked by one of the classic triggers. Examples of such triggers include hemolysis, fasting physical exertion, febrile illness stress, and fatigue. Presumptive diagnosis can be made when the unconjugated hyperbilirubinemia persists with repeat testing but liver function tests complete blood count, blood smear and reticulocyte count are normal. (Choice B) Individuals with the rare Dubin-Johnson syndrome have predominantly conjugated chronic hyperbilirubinemia that is not associated with hemolysis. For the diagnosis to be made conjugated hyperbilirubinemia with a direct bilirubin fraction of at least 50% and an otherwise normal liver function profile must be present. (Choices C and D) Expected laboratory findings in a patient with acute viral hepatitis include significant elevations in ALT and AST (with ALT> AST) followed by rises in bilirubin and alkaline phosphatase. (Choice E) Alcoholic hepatitis is typically associated with a significant drinking history and is commonly characterized by an AST:ALT ratio greater than 2:1. (Choice F) Wilson’s disease is the likely diagnosis in a patient younger than 30 years old with unexplained chronic hepatitis (elevated AST and AST). The presence of low

238

Q NO 43: A 21-year-old Caucasian male presents to your office with mild jaundice after a hiking trip. He had a similar episode two years ago after fasting for five days. His physical examination is unremarkable except for mild jaundice.

Page 59: Patho Physio 50q

USMLE WORLD STEP 1 PATHOPHYSIOLOGY

serum ceruloplasmin and increased urinary copper excretion or Kayser-Fleischer rings provides diagnostic confirmation.

Educational Objective: Gilbert syndrome is the likely diagnosis in patients with no apparent liver disease who have mild unconjugated hyperbilirubinemia that appears provoked by one of the classic triggers.

239

Page 60: Patho Physio 50q

USMLE WORLD STEP 1 PATHOPHYSIOLOGY

A. AB. BC. CD. DE. EF. F

Explanation:The patient gives a history consistent with recurrent temporary arrhythmias, and we come to suspect that these maybe due to an accessory AV conduction pathway (a bundle of Kent). The presence of such a pathway would allow recurrent temporary tachyarrhythmias due to an atrioventricular re-entry circuit involving the AV node and the accessory pathway. The diagram below depicts the most common anatomical anomaly responsible for Wolff Parkinson-White (WPW) pre-excitation syndrome and the most common re-entry circuit responsible for paroxysmal narrow ORS complex (“supraventricular”) tachycardia in patients with WPW.

In patients with this anatomical abnormality when there is no re-entrant tachycardia, normal sinus impulses reach the ventricles first via the accessory pathway and shortly thereafter via the AV node. The accessory pathway “pre-excites” the ventricles, hence the term “pre-excitation syndrome.” The result is a shortened PR interval (often less than 12 seconds) with an early hump (delta wave) at the start of each ORS complex. The ORS complex is wider than normal as well. A demonstrative EGG is pictured below:

240

Q NO 44: A 21-year-old Caucasian male presents to your office complaining of repeated episodes of palpitations that start and stop abruptly. After completing a careful work-up, you believe that there is an abnormal muscle tract in this patient’s heart that bypasses the AV node. If your diagnosis is correct, which of the following parts of the patient’s ECO is most likely to be affected at baseline?

Page 61: Patho Physio 50q

USMLE WORLD STEP 1 PATHOPHYSIOLOGY

(Choice A) Except during periods of tachycardia, a patient with an accessory AV conduction pathway should have a normal p-wave. (Choice B) There are no s-segment changes in WPW syndrome. (Choice C and D) The ST-segment (choice C) corresponds roughly to the plateau phase of the ventricular cardiac myocyte action potential and the T-wave (choice D) corresponds roughly to the repolarization phase of ventricular myocardiocytes action potentials. Due to changes in the normal geographic sequence of ventricular depolarization and repolarization in isolated ventricular pre-excitation, there may be variable ST segment T wave abnormalities, but these are less consistently found than the usual WPVV triad of ECG abnormalities. (Choice F)The QT-interval might theoretically be prolonged to the same extent that the ORS interval is prolonged in a patient with an accessory AV conduction pathway. However, any QT-interval changes are not seen as consistently as the WPW triad of ECG abnormalities.

Educational Objective: An accessory AV conduction pathway can manifest clinically as recurrent paroxysmal supraventricular tachycardia (PSVT) in an otherwise healthy individual. The baseline ECG (i.e. not in the setting of PSVT) of a patient with this anatomy generally shows a triad of abnormalities, which correspond to the ventricular pre-excitation physiology (Wolff Parkinson-White syndrome): a shortened PR-interval, a delta wave at the start of the ORS complex, and a widened QRS- interval.

241

Page 62: Patho Physio 50q

USMLE WORLD STEP 1 PATHOPHYSIOLOGY

A. Endorphin B. Transforming growth factor — beta (TGF—β) C. Brain natriuretic peptide D. Bradykinin E. Endothelin F. Angiotensin II

Explanation:Nesiritide is a recombinant form of human Brain Natriuretic peptide (BNP) and can be used in patients with decompensated left ventricular dysfunction leading to congestive heart failure. In heart failure, especially systolic dysfunction, increased blood volume within the heart causes stretching of the atria and ventricles beyond the appropriate stretch to cause maximal contraction by the Prank-Starling mechanism. This increased stretch causes the release of natriuretic peptides from the walls of the atria (atrial natriuretic peptide, ANP) and the walls of the ventricles (Brain Natriuretic Peptide, BNP). Both ANP and BNP activate guanylate cyclase, which induces an increase of intracellular cyclic GMP. Natriuretic peptides cause vasodilatation, diuresis/natriuresis (as their name implies) and a decrease in blood pressure. They counteract endothelin, sympathetic effects and angiotensin II.

(Choice A) Endorphin, as indicated by the name, is an endogenous substance that acts on the same receptors as narcotic analgesics (ENDO genous mORPHINe). Itis released by the hypothalamic-pituitary axis and does not play a role in diuresis or vasodilatation. Itis synthesized in and released from primarily corticotroph cells in the anterior pituitary. (Choice B)TGP-3 is a molecule synthesized by most cells in the body. Actions of TGF-43 include arrest of the cell cycle (leading to its action as a tumor suppressing agent), promotion of angiogenesis (leading to its action as an agent allowing tumor metastases to survive after they become resistant to TGF), and stimulation of fibroblasts to lay down extracellular matrix proteins (leading to its implication in atherosclerosis and fibrotic diseases). It does not cause the effects mentioned in the question stem. (Choice D) Bradykinin is a hormone produced by the kidney in situations where the adrenergic and renin-angiotensin aldosterone systems are stimulated. It acts locally to constrict veins and dilate arterioles to increase renal perfusion. It is metabolized by ACE and has been implicated as one of the causes of angioedema. This is why patients on ACE inhibitors are predisposed to angioedema. (Choices E and F) Endothelin and Angiotensin II both have effects that are opposite those of brain natriuretic peptide and opposite those presented in the question stem. They are both potent vasoconstrictors that have the effect of increasing the afterload which the failing heart has to pump against in the setting of heart failure. It is believed that endothelin release is mediated by angiotensin II, and this is why the ACE-I class of drugs has been found to be useful in treating heart failure.

242

Q NO 45: A new drug developed for the treatment of congestive heart failure demonstrates favorable effects in both animal experiments and human studies. The drug dilates arterioles and veins and promotes diuresis. The drug described above is most likely an analog of which of the following endogenous substances?

Page 63: Patho Physio 50q

USMLE WORLD STEP 1 PATHOPHYSIOLOGY

Educational Objective: Brain Natriuretic Peptide (BNP) is elevated in patients with heart failure and is often used as a laboratory test in the clinical setting to determine if a patient is suffering from a CHE exacerbation. Itis released by the ventricles when they are stretched as they often are in CHE from systolic dysfunction. It acts along with ANP to cause vasodilatation (decreased preload) and diuresis. Both ANP and BNP activate guanylate cyclase, which induces an increase of intracellular cyclic GMP.

243

Page 64: Patho Physio 50q

USMLE WORLD STEP 1 PATHOPHYSIOLOGY

A.

Acetylcholine B. Serotonin C. Somatostatin D. Gastrin E. Secretin F. Transforming growth factor - α

Explanation:Parietal cells are located in the mucosal glands of the fundus and body of the stomach. They can be easily spotted due to their eosinophilic cytoplasm on H&E stain. These cells have abundant mitochondria and an intracellular tubulovesicular system. They secrete gastric acid (HO) and intrinsic factor. Parietal cells are influenced by a number of substances. Histamine acetylcholine, and gastrin increase gastric acid secretion while PGE2 inhibits it. Gastrin not only facilitates HCI secretion, but it also has atrophic affection parietal cells, causing proliferation and hyperplasia of these cells. In patients with Zollinger-Ellison syndrome (such as in the patient in the vignette), excessive gastrin is produced by tumor cells. The excess gastrin increases gastric acid secretion and induces parietal cell proliferation. Increased acid production leads to peptic ulcer disease and diarrhea. Parietal cell hyperplasia increases the mass of fundic glands, visibly enlarging the fundic mucosal folds. (Choice A) Acetylcholine is a universal neurotransmitter that is present in parasympathetic postganglionic synapses and in all preganglionic synapses. Cholinergic receptors are divided into muscarinic (located in smooth muscle and in glands) and nicotinic (at the neuromuscular junctions). (Choice B) Serotonin (5HT) is found in GIT, brain, platelets retina and other tissues. This mediator helps regulate secretion peristalsis, vomiting and control of emotions by the limbic system. (Choice C) Somatostatin (growth hormone-inhibiting hormone) is secreted by D cells of pancreatic islets and 01 mucosa. Somatostatin inhibits gastrin secretion pancreatic exocrine secretion, gastric secretion and motility, gallbladder contraction and the absorption of all nutrients. (Choice E) Secretin is produced by S cells of the small intestine. It increases bicarbonate production by the pancreas and leads to the secretion of watery, alkaline pancreatic juice. This substance also inhibits gastric acid secretion and stimulates pyloric sphincter contraction. (Choice F)The predominant sources of TGE-α are carcinomas but this chemical is also secreted by macrophages and epithelial cells. TGE-α is a potent stimulator of epithelial growth. In the stomach it causes mucosal-cell hyperplasia, but has little effect on parietal cells.

Educational Objective: Gastrin increases gastric acid secretion and stimulates parietal cell proliferation, thus increasing parietal cell mass (trophic effect). Both these effects are seen in patients with Zollinger-Ellison syndrome.

244

Q NO 46: Partial gastrectomy is performed on a 32-year-old Caucasian male because of refractory peptic ulcer disease. The pathologist receives the tissue and notes significant enlargement of the gastric rugal folds on gross examination. Microscopy of the gastric mucosa reveals parietal cell hyperplasia. Which of the following stimuli is the most likely cause of parietal cell proliferation in this patient?

Page 65: Patho Physio 50q

USMLE WORLD STEP 1 PATHOPHYSIOLOGY

A. Pancreatic ducts B. Small airways C. Sweat ducts D. Vas deferens E. Bile ductules

Explanation:Cystic fibrosis (CF) results from an autosomal recessive mutation in the CFTR gene. Effects on ion and water transport are tissue-specific. In most exocrine epithelia, a normal CFTR will increase chloride conductance (thereby enhancing chloride reabsorption or secretion depending upon local transmembrane electrochemical gradients) and inhibit apical sodium resorption. In sweat ducts, however, the CFTR normally increases luminal membrane sodium resorption. Thus CFTR mutation in sweat gland ducts decreases both sodium and chloride resorption, causing the sweat to be abnormally hypertonic in CF patients. (Choice A) In pancreatic ducts and most other exocrine ducts, the CFTR normally functions to promote chloride secretion and inhibit sodium resorption. The CFTR also increases bicarbonate secretion. Mutation of the CFTR would cause an abnormally high rate of net ion and water resorption, causing isotonic dehydration of the lumen and contributing to ductal obstruction by thickened mucus. In CF patients the abnormal acidity of the pancreatic duct fluid may also contribute to obstruction by promoting mucin precipitation. (Choice B) In the mucous glands of small pulmonary airways, the CFTR normally promotes chloride secretion and inhibits sodium resorption by the mucous gland ductal epithelium. Thus, a CFTR mutation would cause an abnormally high rate of net ion and water resorption, producing isotonic dehydration of the inner luminal duct surface and thickened mucus. In small airways, this viscous mucus can cause airway obstruction and promote infection secondary to mucus plugging and impaired mucociliary clearance. Histologic examination shows bronchioles distended with thick mucus and bronchiectasis. (Choice D) Mutation of the CFTR would cause isotonic dehydration of the contents of the vas deferens as well. However this choice can also be eliminated based on the fact that congenital bilateral absence of the vas deferens is commonly associated with CFTR mutations. (Choice E) Involvement of the bile duct epithelium by a CFTR mutation would cause isotonic dehydration of the luminal contents as in most exocrine glands. Bile canaliculi become plugged by mucinous material potentially resulting in cholestasis, hepatic steatosis, and biliary cirrhosis.

Educational Objective: Mutations in the CFTR gene cause isotonic dehydration of the luminal contents of most exocrine glands. Sweat glands are the exception — here CFTR mutations produce hypertonic luminal fluid containing elevated concentrations of sodium and chloride.

245

Q NO 47: An 8-year-old Caucasian male with recurrent pulmonary infections and otitis media is found to completely lack the cystic fibrosis transmembrane conductance regulator (CETR) on epithelial surfaces. The contents of which of the following lumens is most likely to be hypertonic?

Page 66: Patho Physio 50q

USMLE WORLD STEP 1 PATHOPHYSIOLOGY

A. Lower extremity edema B. Altered mental status C. Palmar erythema D. “Musty” body odor E. Splenomegaly F. Esophageal varices

Explanation:This patient has numerous cirrhotic stigmata that are consistent with cirrhosis of the liver, including gynecomastia, palmar erythema, spider angiomata, asterixis, ascites, pedal edema hepatic encephalopathy, and splenomegaly. Cirrhosis of any type results in the progressive loss of liver functionality. In the cirrhotic patient, gynecomastia arises from hyperestrinism secondary to the damaged liver’s inability to metabolize circulating estrogens. Other manifestations of hyperestrinism in the cirrhotic patient include palmar erythema, spider angiomas, and in males, testicular atrophy and decreased body hair. (Choice A) The liver is the primary site of protein synthesis, and the cirrhotic liver typically produces insufficient amounts of important proteins such as albumin. Hypoalbuminemia leads to a decrease in the intravascular oncotic pressure. Fluid then moves into the extravascular space, and pitting edema in the lower extremities (eg, pedal edema) results. (Choices B and D) Altered mental status and musts body odor (fetor hepaticus) are signs of poor hepatic function and hyperammonemia. In the healthy liver, ammonia is detoxified to urea, which is then excreted in the urine. Because the cirrhotic liver has lost this detoxification ability, ammonia accumulates in the blood. Impaired neuronal function and cerebral edema frequently ensue. (Choices E and F) Portal hypertension arises from increased hepatic resistance to portal flow at the sinusoids and causes increased pressure at the portosystemic collateral veins in the anterior abdomen lower rectum and the lower end of esophagus where the portal and systemic circulations meet. This phenomenon is responsible for the esophageal varices, hemorrhoids, caput medusae, and splenomegaly commonly seen in cirrhotic patients.

Educational Objective: Hyperestrinism in cirrhosis leads to gynecomastia, testicular atrophy, decreased body hair, spider nevi and palmar erythema.

246

Q NO 48: A 45-year-old alcoholic male with history of hepatitis C infection and HIV is brought to the hospital with abdominal distention and altered mental status. He has a “musty” body odor. Physical examination shows gynecomastia, palmar erythema, spider angiomata, and asterixis. Abdomen is extremely distended with dilated periumbilical veins; 3 + bilateral lower extremity edema is present. Abdominal imaging shows splenomegaly. In this patient which of the following has similar pathogenicity of gynecomastia?

Page 67: Patho Physio 50q

USMLE WORLD STEP 1 PATHOPHYSIOLOGY

Chemistry panel

Serum sodium 136 mEq/LSerum potassium 4.0 mEq/LBlood urea nitrogen (BUN) 20 mg/dLSerum creatinine 0.8 mg/dLCalcium 14.0 mg/dL

A bone scan is negative. Which of the following is most likely to be elevated in this patient?

A. Serum phosphorous B. Parathyroid hormone level C. Parathyroid hormone related peptide D. 25 hydroxyvitamin D level E. Serum prolactin level

Explanation:This long-term smoker has lung mass and respiratory symptoms that probably represent cancer; and smoking is a very significant risk factor for squamous cell lung cancer. This patient’s labs also demonstrate elevated serum calcium. Answering this question will depend on your knowledge of a paraneoplastic phenomenon known as the “humoral hypercalcemia of malignancy.” This hypercalcemia is caused by factors usually secreted by non-metastatic cancers. Although other factors maybe responsible parathyroid hormone-related peptide (PTHrP) is by far the most significant cause of the humoral hypercalcemia of malignancy. The structure of PTHrP closely resembles parathyroid hormone (PTH) at the bioactive amino-terminal region. As a result, PTHrP acts similarly to PTHI although the degree of hypercalcemia is generally higher with PTHrP production than in primary hyperparathyroidism. (Choice A) As a rule of thumb if calcium levels are high, phosphorous levels are low, and vice versa. Phosphorous is certain to be low in this patient because PTHrP is a potent inhibitor of phosphorus reabsorption from the proximal renal tubule. (Choice B) PTHrP causes hypercalcemia, which suppresses the secretion of PTH via calcium’s negative feedback. (Choice D) Serum levels of 25-hydroxvvitamin D reflect the oral intake and endogenous formation of vitamin D. Neither PTHrP nor PTH have an effect on the 25-hydroxylation step in the liver. (Choice E) Serum prolactin level is not related to this patient’s condition.

Educational Objective: The hypercalcemia associated with squamous cell lung cancer and several other tumors, is usually due to overproduction of parathyroid hormone-related peptide and is termed the “humoral hypercalcemia of malignancy.”

247

Q NO 49: A 65-year-old long-time smoker is evaluated for shortness of breath and cough. Chest X-ray shows a mass in the right upper lobe. Laboratory studies return with the following values:

Page 68: Patho Physio 50q

USMLE WORLD STEP 1 PATHOPHYSIOLOGY

A. Weight gain and easy bruising B. Paresthesia and muscle weakness C. Excessive hair growth D. Sweating and tremulousness E. Diarrhea and flushing

Explanation:The outer layer of the adrenal gland, the zona glomerulosa, normally produces aldosterone. The patient in the vignette most likely suffers from primary hyperaldosteronism (Conn’s syndrome). Aldosterone-secreting adrenal adenomas are the most common cause of Conn’s syndrome (65% of cases). Idiopathic hyperaldosteronism, including primary adrenal hyperplasia, accounts for 30—40% of cases of Conn’s syndrome. Hyperaldosteronism produces renal Na retention and excess K secretion, resulting in hypertension and hypokalemia. Additionally, patients with Conn’s syndrome commonly experience a metabolic alkalosis. The hypokalemia may cause muscle weakness (hypokalemic paresis). Hypokalemic alkalosis may promote paresthesias. (Choice A) Weight gain and easy bruising are symptoms of glucocorticoid excess (Cushing’s syndrome). Easy bruising results from inhibited collagen and matrix glycosaminoglycan synthesis, resulting in thinner, weaker skin and connective tissue. A cortisol-secreting adrenal lesion would have the functionality of the intermediate layer of the three-layered adrenal cortex, the zona fasciculata. (Choice C) Excessive hair growth (hirsutism) can result from androgen excess. An adrenal tumor causing androgen hypersecretion would have the functionality of the third (innermost) layer of the adrenal cortex, the zona reticularis. (Choice D) Sweating and tremulousness can be caused by excess circulating catecholamines, which may result from a pheochromocytoma, the most common tumor of the adrenal medulla in adults. (Choice E) Diarrhea and flushing may be seen in carcinoid syndrome, a rare condition caused most often by a serotonin (5-HT)-secreting tumor derived from neuroendocrine cells of the gastrointestinal tract.

Educational Objective: If a hypersecreting-adrenal tumor has the functionality of the outer, intermediate, or inner layers of the adrenal cortex, the hormone(s) released and clinical syndrome produced will be aldosterone/Conn’s syndrome, cortisol/Cushing’s syndrome, and androgens/hirsutism and virilization, respectively.

248

Q NO 50: A 45-year-old Caucasian female undergoes left-sided adrenalectomy. Gross examination of the gland demonstrates a well-defined yellowish tumor in the adrenal cortex. If the tumor was composed of functional cells from the outer layer of adrenal cortex the patient would most likely complain or